You are on page 1of 36

VISIONIAS

www.visionias.in
ANSWERS & EXPLANATIONS
GENERAL STUDIES (P) TEST – 3790 (2023)

Q 1.A
• The famous Lord Macaulay’s Minute 1835 settled the row in favour of Anglicists against
orientalists and recommended that the limited government resources were to be devoted to teaching
Western sciences and literature. The government soon made English the medium of instruction in its
schools and colleges and opened a few English schools and colleges instead of a large number of
elementary schools, thus neglecting mass education.
• The British planned to educate a small section of upper and middle classes, thus creating a class
“Indian in blood and color but English in tastes, in opinions, in morals and in intellect” who would
act as interpreters between the government and masses and would enrich the vernaculars by which
knowledge of Western sciences and literature would reach the masses. This was called the
‘downward filtration theory. Hence option (a) is the correct answer.
• Modern ideas, if not education, did filter down to the masses, though not in a form desired by the rulers,
but through political parties, press, pamphlets, public platforms, etc. Modern education only helped this
process by making available the basic literature on physical and social sciences to nationalists, thus
stimulating their capacity to make social analysis—otherwise, the content, structure, and curricula of
modern education served colonial interests.

Q 2.C
• Recently, the Indian Computer Emergency Response Team (CERT-In) under the Ministry of Electronics
and Information Technology (MeitY) issued new directives that require Virtual Private Networks
(VPN) service providers to store user data for five years. Hence statement 2 is not correct.
o Under the new directions, VPN providers will need to store validated customer names, their physical
addresses, email ids, phone numbers, and the reason they are using the service, along with the dates
they use it and their “ownership pattern".
o It is aimed to help limit cybercrime and cybersecurity incidents in the country. Failing to furnish the
information or non-compliance with the directions may invite "punitive action" under sub-section (7)
of section 70B of the IT Act, 2000, and other laws as applicable.
• A VPN is an encrypted connection over the Internet from a device to a network that helps users hide
their browsing history, internet protocol (IP) address, geographical location, web activities, and the
devices being used. Hence statement 1 is correct.
• CERT-In is the nodal agency to issue directives for VPN service providers in case of cyber security
threats or to boost the cyber security infrastructure. Hence statement 3 is correct.

Q 3.A
• The catalyst to the new phase of the movement was provided when the British government appointed
Indian Statutory Commission, popularly known after the name of its chairman, as Simon
Commission, to go into the question of further constitutional reforms. All the members of this
commission were Englishmen. This announcement was greeted by a chorus of protest from all Indians.
Hence statement 1 is correct.
• The call for a boycott of the Commission was endorsed by the Liberal Federation led by Tej
Bahadur Sapru. The Hindu Mahasabha and the section of the Muslim League under Jinnah supported the
call for boycott. It was the Indian National Congress, however, that turned the boycott into a popular
movement. The Congress had resolved on the boycott at its annual session in December 1927 at
Madras, and in the prevailing excitable atmosphere, Jawaharlal Nehru had even succeeded in getting
passed a snap resolution declaring complete independence as the goal of the Congress.
1 www.visionias.in ©Vision IAS
• The Simon Commission published a two-volume report in May 1930. It proposed the abolition of dyarchy
and the establishment of representative government in the provinces which should be given autonomy.
The report contained no mention of Dominion Status. Hence statement 2 is not correct.

Q 4.D
• Queen Victoria's Proclamation of 1 November 1858 declared that thereafter India would be governed by
and in the name of the British Monarch through a Secretary of State.
• The assumption of the Government of India by the sovereign of Great Britain was announced by Lord
Canning at a durbar at Allahabad in the ‘Queen’s Proclamation’ issued on November 1, 1858.
• It was by this proclamation that the governor-general acquired the additional title of ‘Viceroy’.
Hence statement 1 is correct.
• Many of the promises made in that proclamation appeared to be of a positive nature to the Indians. As per
the Queen’s proclamation, the era of annexations and expansion had ended and the British promised to
respect the dignity and rights of the native princes.
• The Indian states were henceforth to recognize the paramountcy of the British Crown and were to
be treated as parts of a single charge. Hence statement 2 is correct.
• The people of India were promised freedom of religion without interference from British officials.
• The proclamation also promised equal and impartial protection under the law to all Indians,
besides equal opportunities in government services irrespective of race or creed. Hence statement 3
is correct.
• It was also promised that old Indian rights, customs, and practices would be given due regard while
framing and administering the law.

Q 5.A
• The failure of the Cripps Mission paved the way for a full-fledged confrontation between the British
and the Congress. As a result Congress announced the Quit India campaign in August 1942
and refused to help the British in the war effort, and the British imprisoned the entire Congress
leadership in return. Hence statement 1 is correct.
• The Quit India Movement made clear that now there was no distinction to be made between the
people of British India and the States: every Indian was to participate in this mass struggle The meeting
of the All India State Peoples Conference (AISPC) was convened along with the Congress session at
Bombay that announced the commencement of the Movement. Hence statement 2 is correct.
• The Communist Party of India, following the involvement of Russia in the war in December 1941, did
not support Quit India movement because of their “Peoples’ War” Strategy, Other parties like
Muslim League, Hindu Mahasabha were also against it. Hence statement 3 is not correct.

Q 6.A
• The members of the Constituent Assembly were chosen on the basis of Provincial elections of 1946.
Apart from the members sent by the provinces of the British India, it also had representatives from the
Princely States and the Muslim League chose to boycott the early sittings making it effectively a one-
party show as 82 per cent of the members of the Assembly were members of the Congress Party.
Hence statement 1 is correct.
• “The Objectives Resolution” was introduced by Jawaharlal Nehru in the very first session of the
Constitution Assembly; these resolutions were the considered as the aspirations of constitutional
forefathers and it is the preamble of aspirations, which the constitution makers sought to achieve after the
independence in the form of a new India. And these resolutions were framed as the Preamble of the
Indian Constitution. Hence statement 2 is not correct.

Q 7.A
• The Pitt’s India Act of 1784 gave the British government a large measure of control over the Company’s
affairs. The Company became a subordinate department of the State. The Company’s territories in India
were termed ‘British possessions’. The act clearly subordinated the Bombay and Madras presidencies to
Bengal in all questions of war, diplomacy, and revenues.
• It constituted a department of state in England, also known as the Board of Control, whose purpose
served to control the policy of the Court of Directors, introducing the Dual System of Government.
• The government’s control over the Company’s affairs was greatly extended. A Board of Control
consisting of the chancellor of the exchequer, a secretary of state, and four members of the Privy Council
(to be appointed by the Crown) were to exercise control over the Company’s civil, military, and revenue
2 www.visionias.in ©Vision IAS
affairs. All dispatches were to be approved by the board. Thus, a dual system of control was set up. Hence
statements 1 and 2 are correct.
• Despite these changes, Company was satisfied as the act saved its monopoly over Indian and
Chinese trade. Its directors retained the profitable right of appointing and dismissing its officials in
India. Hence statement 3 is not correct.

Q 8.B
• The 'Swadesamitran' was the first Tamil newspaper published in India. This newspaper had the largest and
the most widespread circulation of any Tamil publication of the Madras Presidency. It had maximum
circulation in other countries also such as in Upper and Lower Burma, Ceylon, Penang, Singapore.
o The Swadesamitran was established in 1882 and was run by G. Subramania Iyer, a veteran
journalist of South India.T N Viswanathan skillfully assisted him in the running of this newspaper.
The largest advertisers in England and America also preferred the Swadesamitran as a medium for
advertisements. Hence pair 1 is correctly matched.
• The legendary Amrita Bazar Patrika and its journalists were at the forefront of India’s struggle for
Independence, and early proponents of the concept of press freedom. Many even claim that the
Vernacular Press Act was mainly targeted at the Amrita Bazar Patrika since its sharp criticism of the
British was making the colonial government uncomfortable.
o Amrita Bazar Patrika was first published on 20 February 1868 by two brothers Sisir and Motilal
Ghosh who belonged to a rich Bengali merchant family.
• Keshab Chandra Sen started the Indian Mirror as a fortnightly in 1861 under the patronage
of Debendra Nath Tagore, with renowned barrister Manomohan Ghosh as editor. Narendra Nath Sen
(1843-1911) was a regular contributor in the paper. He was offered to edit the fortnightly in 1863. In
1867, Keshab Sen requested Brahmo leader Protap Mozoomdar to take charge of the paper. Mozoomdar
complied and continued as editor till 1879. After he left, Sen returned but this time he became its sole
proprietor and editor. Meanwhile, The Indian Mirror had become a daily since 1870. Hence pair 2 is
correctly matched.
• Voice of India newspaper was started by Dadabhai Naoroji. Dadabhai Naoroji was an Indian social-
political leader and one of the founders of the Indian National Congress. A leading nationalist author and
spokesman, he was the first Indian to be elected to membership in the British Parliament. Hence pair
3 is not correctly matched.

Q 9.A
• Theosophical Movement:
o A group of westerners led by Madame H.P. Blavatsky (1831- 1891) and Colonel M.S. Olcott, who
was inspired by Indian thought and culture, founded the Theosophical Society in New York City,
United States in 1875.
o In 1882, they shifted their headquarters to Adyar, on the outskirts of Madras (at that time) in
India. Hence statement 1 is correct.
o Society believed that a special relationship could be established between a person’s soul and God by
contemplation, prayer, revelation, etc.
o It accepted the Hindu beliefs in reincarnation and karma and drew inspiration from the
philosophy of the Upanishads and Samkhya, yoga, and Vedanta schools of thought. Hence
statement 2 is not correct.
o It aimed to work for the universal brotherhood of humanity without distinction of race, creed, sex,
caste, or colour. The society also sought to investigate the unexplained laws of nature and the powers
latent in man.
o The Theosophical Movement came to be allied with the Hindu renaissance. (At one time it allied with
the Arya Samaj too.) It opposed child marriage and advocated the abolition of caste discrimination,
uplift of outcastes, and improvement in the condition of widows.
o In India, the movement became somewhat popular with the election of Annie Besant (1847-1933) as
its president after the death of Olcott in 1907.
Q 10.B
• The viceroy, Lord Minto, and the Secretary of State for India, John Morley worked out a set of measures
that came to be known as the Morley- Minto (or Minto-Morley) Reforms that translated into the Indian
Councils Act of 1909.
• Morley openly declared in Parliament: ‘If it could be said that this chapter of reforms led directly or
necessarily up to the establishment of a Parliamentary system in India, I, for one, would have nothing at
all to do with it.’
3 www.visionias.in ©Vision IAS
• The real purpose of the Morley-Minto Reforms was to divide the nationalist ranks and to check the
growing unity among Indians by encouraging the growth of Muslim communalism.
• Whereas, Edwin Montagu who served as Secretary of State for India between 1917 and 1922, on
20th August 1917, made a historic declaration in the House of Commons defining the goal of British
policies in India. The declaration stated: ‘The policy of His Majesty’s Government is that of
the increasing association of Indians in every branch of the administration, and the gradual
development of self-government institutions, with a view to the progressive realisation of
responsible government in India as an integral part of the British Empire'. Hence statement 1 is not
correct.
• Most of the elected members were still elected indirectly. An Indian was to be appointed a member of the
Governor-General’s Executive Council. Of the sixty-eight members of the Imperial Legislative Council,
thirty-six were officials, and five were nominated non-officials.
o Out of twenty-seven elected members, six were elected by big landlords and two by British
capitalists. Hence statement 2 is correct.
• The real purpose of the Morley-Minto Reforms was to divide the nationalist ranks and to check the
growing unity among Indians by encouraging the growth of Muslim communalism. To achieve the latter
objective, the Reforms introduced the system of separate electorates under which Muslims could
only vote for Muslim candidates in constituencies specially reserved for them. This was done to
encourage the notion that the political, economic, and cultural interests of Hindus and Muslims
were separate and not common. Hence statement 3 is correct.

Q 11.C
• Annie Besant, already sixty-six in 1914, had begun her political career in England as a proponent of Free
Thought, Radicalism, Fabianism, and Theosophy, and had come to India in 1893 to work for the
Theosophical Society. Hence statement 1 is correct.
• In early 1915, Annie Besant launched a campaign through her two papers, New India and
Commonweal, and organized public meetings and conferences to demand that India be granted self-
government on the lines of the White colonies after the War. From April 1915, her tone became more
peremptory and her stance more aggressive. Hence statement 2 is correct.
• Annie Besant had gone ahead with the formal founding of her League in September 1916. Most of the
work was carried on by Annie Besant and her lieutenants — Arundale, C.P. Ramaswamy Aiyar, and B.P.
Wadia — from her headquarters at Adyar. Hence statement 3 is not correct.
• Annie Besant was the first woman President of the Indian National Congress. She presided over the
1917 Calcutta session of the Indian National Congress. The president of the 1916 Lucknow session was
Ambica Charan Mazumdar. Hence statement 4 is correct.

Q 12.D
• The British desire to extend their Indian empire to its natural geographical frontier brought them into
conflict with Kingdom of Nepal. In October 1814 a border clash between the border police of two
countries led to open war.
• The war ended in the Treaty of Sagauli in 1816, which was in favour of the British. As per the treaty :
o Nepal accepted a British resident.
o Nepal ceded the districts of Garhwal and Kumaon, and abandoned claims to Terai.
o Nepal also withdrew from Sikkim.
o It ceded the districts of Garhwal and Kumaon and abandoned claims to Tarai areas.
• This agreement brought many advantages to the British:
o British empire now reached the Himalayas;
o It got better facilities for trade with Central Asia
o It acquired sites for hill stations, such as Shimla, Mussoorie, and Nainital; and the Gorkhas joined the
British Indian Army in large numbers.
• Hence, option (d) is the correct answer.

Q 13.C
• Sasipada Banerjee was a social worker and leader of the Brahmo Samaj who is remembered as a
champion of women's rights and education and as one of the earliest workers for labor welfare in India.
He was the founder of several girls' schools, a widow's home.
• Banerjee became involved in the social reform movement in Bengal through the Brahmo Samaj which he
joined in 1861.

4 www.visionias.in ©Vision IAS


• He set up a Workingmen’s Club in 1870 and brought out a monthly journal called Bharat
Sramjeebi (Indian Labour), with the primary idea of educating the workers.
• Banerjee was a member of the Temperance movement in India and was a close associate of Mary
Carpenter whom he first met during her visit to India in 1866.
• Hence, option (c) is the correct answer.

Q 14.B
• Recent Context: The Ministry of Finance has revised the Member of Parliament Local Area
Development Scheme (MPLADS) rules, where the interest that the fund accrues will be deposited in
the Consolidated Fund of India. So far, the interest accrued on the fund used to be added to the
MPLADS account and could be used for the development projects. Hence statement 2 is correct.
• MPLADS is Central Sector Scheme that was announced in December 1993. The objective is to enable
MPs to recommend works of developmental nature with emphasis on the creation of durable community
assets in the areas of drinking water, primary education, public health, sanitation, roads, etc. primarily in
their Constituencies.
• Each year, MPs receive Rs. 5 crores in two installments of Rs. 2.5 crore each. Funds under MPLADS
are non-lapsable. Lok Sabha MPs have to recommend the district authorities' projects in their Lok Sabha
constituencies, while Rajya Sabha MPs have to spend it in the state that has elected them to the
House. Nominated Members of both the Rajya Sabha and Lok Sabha can recommend works
anywhere in the country. Hence statement 1 is correct.
• The policy formulation and implementation of the MPLADS scheme are done by the Ministry of
Statistics and Programme Implementation, Hence statement 3 is not correct.

Q 15.B
• Indian newspapers began to find their feet in the 1870s. They became highly critical of Lord Lytton’s
administration, especially regarding its inhuman approach towards the victims of the famine of 1876-77.
• As a result, the Government decided to make a sudden strike at the Indian language newspapers, since
they reached beyond the middle-class readership. The Vernacular Press Act of 1878 was directed only
against Indian-language newspapers. It was conceived in great secrecy and passed at a single sitting of
the Imperial Legislative Council. Hence, statement 1 is correct.
• The Act provided for the confiscation of the printing press, paper and other materials of a newspaper if
the Government believed that it was publishing seditious materials and had flouted an official
warning. Hence, statement 2 is correct.
• Indian nationalist opinion firmly opposed the Act. The first great demonstration on an issue of public
importance was organized in Calcutta on this question when a large meeting was held in the Town Hall.
Various public bodies and the Press also campaigned against the Act. Consequently, it was repealed in
1881 by Lord Ripon. Hence, statement 3 is not correct.

Q 16.B
• Statement 1 is not correct: At the end of December 1929, Congress held its annual session in the city of
Lahore. The meeting was significant for the election of Jawaharlal Nehru as President, signifying the
passing of the baton of leadership to the younger generation. Only three out of eighteen Provincial
Congress Committees had wanted Jawaharlal Nehru as president but recognizing the appositeness of the
occasion, and the upsurge of the youth who had made such a glorious success of the Simon Boycott,
Gandhiji supported and was decisive in electing Nehru as the President.
• Statement 2 is correct: The first task that the Congress set itself and the Indian people in the New Year
was that of organizing all over the country, on 26 January, public meetings at which the Independence
Pledge would be read out and collectively affirmed. This programme was a huge success, and in villages
and towns, at small meetings and large ones, the pledge was read out in the local language and the
national flag was hoisted. The Lahore Congress of 1929 had authorized the Working Committee to
launch a programme of civil disobedience including non-payment of taxes. It had also called upon all
members of legislatures to resign their seats.
• Statement 3 is correct: Jawaharlal Nehru, Subhas Bose and Satyamurthi, backed by a large number of
delegates, pressed for the acceptance of ‘Purna Swaraj’ or complete independence as the goal of the
Congress and on 19 December 1929, INC passed the historic 'Purna Swaraj' - (total independence)
resolution.

5 www.visionias.in ©Vision IAS


Q 17.B
• One of the most militant and widespread of the peasant movements was the Indigo Revolt of 1859-60.
From the beginning, indigo was grown under an extremely oppressive system that involved great loss to
the cultivators. The indigo planters, nearly all Europeans, compelled the tenants to grow indigo
which they processed in factories set up in rural (mofussil) areas. The planters forced the peasants to
take a meager amount as advance and enter into fraudulent contracts.
• Since the enforcement of forced and fraudulent contracts through the courts was a difficult and prolonged
process, the planters resorted to a reign of terror to coerce the peasants. Kidnapping, illegal
confinement in factory godowns, flogging, attacks on women and children, carrying off cattle, looting,
were some of the methods used by the planters. They hired or maintained bands of lathyals (armed
retainers) for the purpose. Hence, statement 1 is not correct.
• In practice, the planters were also above the law. With a few exceptions, the magistrates, mostly
European, favoured the planters. The discontent of indigo growers in Bengal boiled over in the autumn of
1859 when their case seemed to get Government support. Misreading an official letter and exceeding his
authority, Hem Chandra Kar, Deputy Magistrate of Kalaroa, published on 17th August a proclamation to
policemen that ‘in case of disputes relating to Indigo Ryots, they (ryots) shall retain possession of their
own lands and shall sow on them what crops they please, and the Police will be careful that no Indigo
Planter nor anyone else is able to interfere in the matter. The news of Kar’s proclamation spread all over
Bengal, and peasants felt that the time for overthrowing the hated system had come. The beginning was
made by Digambar Biswas and Bishnu Biswas, ex-employees of a planter, they gave up indigo
cultivation.
• The peasant disturbances and indigo strikes spread rapidly to other areas. The peasants refused to take
advances and enter into contracts, pledged not to sow indigo, and defended themselves from the planters’
attacks with whatever weapons came to hand. The planters then attacked with another weapon, their
zamindari powers. They threatened the rebellious ryots with eviction or enhancement of rent. The ryots
replied by going on a rent strike. Peasants gradually learnt to use the legal machinery to enforce their
rights. They also used the weapon of social boycott to force a planter’s servants to leave him. Ultimately,
the planters could not withstand the united resistance of the ryots, and they gradually began to close their
factories. The cultivation of indigo was virtually wiped out from the districts of Bengal by the end of
1860.
• A major reason for the success of the Indigo Revolt was the tremendous initiative, cooperation,
organization and discipline of the ryots. Another was the complete unity among Hindu and Muslim
peasants. Leadership for the movement was provided by the more well-off ryots and in some cases by
petty zamindars, moneylenders and ex-employees of the planters.
• The intelligentsia’s role in the Indigo Revolt was to have an abiding impact on the emerging nationalist
intellectuals very political childhood they had given support to a popular peasant movement against the
foreign planters. This was to establish a tradition with long-run implications for the national
movement. Christian Missionaries were another group that extended active support to the indigo
ryots in their struggle. Hence, statement 2 is correct.
• Worried by the rebellion, the government brought in the military to protect the planters from assault, and
set up the Indigo Commission to enquire into the system of indigo production. The Commission held the
planters guilty, and criticised them for the coercive methods they used with indigo cultivators. It
declared that indigo production was not profitable for ryots. Hence statement 3 is correct.
• Hence option (b) is the correct answer.

Q 18.D
Storm Centres and Leaders of the Revolt
• At Delhi the nominal and symbolic leadership belonged to the Mughal emperor, Bahadur Shah, but
the real command lay with a court of soldiers headed by General Bakht Khan who had led the
revolt of Bareilly troops and brought them to Delhi.
• Emperor Bahadur Shah was perhaps the weakest link in the chain of leadership of the revolt. His weak
personality, old age and lack of leadership qualities created political weakness at the nerve centre of the
revolt and did incalculable damage to it.
• At Kanpur, the natural choice was Nana Saheb, the adopted son of the last peshwa, Baji Rao II. He was
refused the family title and banished from Poona, and was living near Kanpur. Nana Saheb expelled the
English from Kanpur, proclaimed himself the peshwa, acknowledged Bahadur Shah as the

6 www.visionias.in ©Vision IAS


Emperor of India and declared himself to be his governor. Sir Hugh Wheeler, commanding the
station, surrendered on June 27, 1857 and was killed on the same day.
• Begum Hazrat Mahal took over the reigns at Lucknow where the rebellion broke out on June 4,
1857 and popular sympathy was overwhelmingly in favour of the deposed nawab. Her son, Birjis
Qadir, was proclaimed the nawab and a regular administration was organised with important offices
shared equally by Muslims and Hindus.
• At Bareilly, Khan Bahadur, a descendant of the former ruler of Rohilkhand, was placed in
command. Not enthusiastic about the pension being granted by the British, he organised an army of
40,000 soldiers and offered stiff resistance to the British.
• In Bihar, the revolt was led by Kunwar Singh, the zamindar of Jagdishpur. An old man in his seventies, he
nursed a grudge against the British who had deprived him of his estates. He unhesitatingly joined the
sepoys when they reached Arrah from Dinapore (Danapur).Hence option (d) is the correct answer.

Q 19.B
• After the withdrawal of the Non-Cooperation Movement in February 1922, a new line of political activity,
which would keep up the spirit of resistance to colonial rule, was advocated by C.R. Das and Motilal
Nehru. They suggested that the nationalists should end the boycott of the legislative councils, enter
them, expose them as ‘sham parliaments’ and as ‘a mask which the bureaucracy has put on,’ and obstruct
‘every work of the council.’
• Statement 1 is correct and statement 2 is not correct: C.R. Das as the President of the Congress and
Motilal as its Secretary put forward this program of ‘either mending or ending’ the councils at the Gaya
session of the Congress in December 1922. Another section of the Congress, headed by Vallabhbhai
Patel, Rajendra Prasad and C. Rajagopalachari, opposed the new proposal which was consequently
defeated. Das and Motilal resigned from their respective offices in the Congress and on 1 January 1923
announced the formation of the Congress-Khilafat Swaraj Party better known later as the Swaraj
Party. The adherents of the council entry program came to be popularly known as ‘pro-changers’ and
those still advocating boycott of the councils as ‘no-changers.’ Das was the President and Motilal was
one of the Secretaries of the new party. The adherents of the council-entry programme came to be
popularly known as ‘pro-changers’ and those still advocating boycott of the councils as ‘no-changers.’
• The no-changers, whose effective head was Gandhiji even though he was in jail, argued for the
continuation of the full programme of boycott and non-cooperation, effective working of the constructive
programme and quiet preparations for the resumption of the suspended civil disobedience. The
Swarajists claimed that they would turn the legislatures into arenas of political struggle and that
their intention was not to use them, as the Liberals desired, as organs for the gradual
transformation of the colonial state, but to use them as the ground on which the struggle for the
overthrow of the colonial state was to be carried out.
• As the pro-changer no-changer clash developed, the atmosphere of dismay in nationalist ranks began to
thicken, and they began to be haunted by the fear of the repetition of the disastrous split of 1907.
Consequently, in a special session of the Congress held at Delhi in September 1923, the Congress
suspended all propaganda against council-entry and permitted Congressmen to stand as candidates
and exercise their franchise in forthcoming elections.

Q 20.A
• Cabinet Mission was a high-powered mission sent in February 1946 to India by the Atlee Government, It
comprised Sir Pethick Lawrence, Secretary of State for India, Sir Stafford Cripps, President of the
Board of Trade, V Alexander, the First Lord of the Admiralty. It was sent to negotiate with the Indian
leaders the terms of the transfer of power to Indians. Hence statement 1 is correct.
• Major recommendations:
o It recommended an undivided India. There shall be a Union of India. It will deal with the defense,
foreign affairs, and communications. It also restricted Communal representation.
o All the members of the Interim cabinet would be Indians and there would be minimum
interference by the Viceroy.
o Formation of the constituent assembly on the democratic principle of population
o It recognized India’s right to cede from the Commonwealth
o There was to be a federation of the provinces and the States, with the federal center controlling
only defense, foreign affairs, and communications. The union would have the powers necessary to
raise the finances to manage the subjects. All subjects other than the Union subjects and all the
residuary powers would be vested in the provinces. Hence statement 2 is correct.

7 www.visionias.in ©Vision IAS


o The Princely states would retain all subjects and all residuary powers.
o The representation of the Provincial legislatures was to be broken up into 3 sections.
 Section A: Madras, UP, Central provinces, Bombay, Bihar & Orissa
 Section B: Punjab, Sindh, NWFP, Baluchistan
 Section C: Assam and Bengal
• Both Indian National Congress and Muslim League accepted this plan. But the two could not agree
on the plan for an interim government that would convene a constituent assembly to frame a constitution
for the free, federal India. Hence statement 3 is not correct.
Q 21.C
• The echoes of Tilak's famous trial and imprisonment in 1908 were to be heard again in 1922.
Mahatma Gandhi, Tilak's political successor, was tried in 1922 for the same offence of sedition
under the same Section 124A for his articles in Young India. Mahatma Gandhi was arrested near
Sabarmati Ashram for writing three articles in Young India.
• When the Judge told him that his offence was similar to Tilak’s and that he was giving him the same
sentence of six years’ imprisonment Gandhiji replied: ‘Since you have done me the honor of recalling the
trial of the late Lokamanya Bal Gangadhar Tilak, I just want to say that I consider it to be proudest
privilege and honor to be associated with his name.”
• The only difference between the two trials was that Gandhiji had pleaded guilty to the charges. This
was also a measure of the distance the national movement had travelled since 1908. He was sentenced to
six years imprisonment. Gandhiji was released from Yervada prison on 5 February, 1924
unconditionally after an operation on 12 January, 1924
• Hence, both statements are correct.

Q 22.B
• Indian Space Research Organisation (ISRO) successfully tests solid rocket booster HS200 for the
Gaganyaan mission.
• It is a 20-meter-long booster with a diameter of 3.2 meters and is one of the largest operational
boosters using solid propellants. Hence statement 1 is not correct.
• The test was held at the Satish Dhawan Space Centre, Sriharikota, It was designed and developed by the
Vikram Sarabhai Space Centre (VSSC) in Thiruvananthapuram over two years.
o It is the human-rated version of the S200 rocket boosters used on the Geosynchronous Satellite
Launch Vehicle Mk-III (GSLV Mk-III), also called the LVM3.
• The GSLV Mk-III rocket which will be used for the Gaganyaan mission will have two HS200
boosters that will supply the thrust for lift-off. Hence statement 2 is corerct.

Q 23.C
• The Indian National Congress (INC) did not take part in either the first Round Table Conference
or the Third Round Table Conference, but it did take part in the Second Round Table Conference
with Mahatma Gandhi as its representative. Hence option (c) is the correct answer.
• The second Round Table Conference was held in London from September 7, 1931 to December 1,
1931. At the conference, Gandhiji claimed that Congress represent all people of India against
imperialism. However, the other delegates did not agree to this view.
o There was a deadlocked on the question of the minorities. Muslims, Depressed classes, Christians and
Anglo-Indians demanded separate electorates. To bolster their demand, they all came together in a
‘Minorities’ Pact’.
o Gandhiji was against the move by the minorities to making all constitutional progress conditional on
the issue of separate electorates. He said that untouchables are hindus and hence, should not be
treated as a minority. Similarly, he discarded the idea of any separate electorate for muslims or any
other minority.
o Princely states were not too enthusiastic about a federation, especially after the possibility of the
formation of a Congress government at the centre with substantial role of elected members.
• Other Facts Regarding Round Table Conference:
o Round Table Conferences (RTCs) were attended by different groups via their representatives, such as
Princely states, Muslim League, Sikhs, Parsis, Women, Liberals, Depressed classes etc.
o Depressed classes were represented by Dr. Bhimrao Ambedkar in all the three round table
conferences (RTCs).
o Similarly, liberals were represented by Tej Bahadur Sapru in all the RTCs.
o While, Begum Jahanara Shahnawaz represented women in every RTC.

8 www.visionias.in ©Vision IAS


Q 24.A
• The third pillar of British rule was the police whose creator was Cornwallis. He relieved the
Zamindars of their police functions and established a regular police force to maintain law and
order.
• In this respect, he went back to and modernised the old Indian system of thanas. This put India ahead of
Britain where system of police had not developed yet.
• He organised a regular police force to maintain law and order by establishing a system of thanas
(circles) in a district under a daroga (an Indian) and a superintendent of police (SP) at the head of a
district.
• Hence, option (a) is the correct answer.

Q 25.C
• The nationalist agitation forced the Government to make some changes in legislative functioning by the
Indian Councils Act of 1892.
• The number of additional members of the Imperial and Provincial Legislative Councils was increased
from the previous six to ten to ten to sixteen.
• A few of these members could be elected indirectly through municipal committees, district boards, etc.,
but the official majority remained.
• The members were given the right to discuss the annual budget but they could neither vote on it nor move
a motion to amend it.
• They could also ask questions but were not allowed to put supplementary questions or to discuss the
answers.
• The ‘reformed’ Imperial Legislative Council met, during its tenure till 1909, on an average for only
thirteen days in a year, and the number of unofficial Indian members present was only five out of twenty-
four.
• The nationalists were totally dissatisfied with the Act of 1892. They saw in it a mockery of their demands.
• The Councils were still impotent; despotism still ruled. They now demanded a majority for non-official
elected members with the right to vote on the budget and, thus, on the public purse.
• They raised the slogan ‘no taxation without representation.’ Gradually, they raised their
demands. Hence, option (c) is the correct answer.

Q 26.D
• The Swadeshi Movement had its genesis in the anti-partition movement which was started to oppose the
British decision to partition Bengal.
o The attempt, at that time in the words of Lord Curzon, the Viceroy (1899-1905) was to
‘dethrone Calcutta’ from its position as the ‘centre from which the Congress Party is
manipulated throughout Bengal, and indeed which the Congress Party centre of successful
intrigue’ and ‘divide, the Bengali speaking population.’
o Risley, the Home Secretary to the Government of India, was more blunt. He said on 6 December
1904: ‘Bengal united, is power, Bengal divided, will pull several different ways.
• In December 1903, the partition proposals became publicly known, and immediate and spontaneous
protest followed. Surendranath Banerjea, Krishna Kumar Mitra, Prithwishchandra Ray and other leaders
launched a powerful press campaign against the partition proposals through journals and newspapers like
the Bengalee, Hitabadi and Sanjibani.
• Vast protest meetings were held in the town hail of Calcutta in March 1904 and January 1905, and
numerous petitions (sixty-nine memoranda from the Dacca division alone), some of them signed by as
many as 70,000 people were sent to the Government of India and the Secretary of State. Even, the big
zamindars who had hitherto been loyal to the Raj joined forces with the Congress leaders who were
mostly intellectuals and political workers drawn from journalism, law and other liberal
professions. Hence statement 1 is not correct.
• The day partition took effect — 16 October 1905 — was declared a day of mourning throughout Bengal.
People fasted and no fires were lit at the cooking hearth.
o In Calcutta, a hartal was declared. People took out processions and band after band walked barefoot,
bathed in the Ganges in the morning and then paraded the streets singing Bande Mataram which,
almost spontaneously, became the theme song of the movement.
• The Indian National Congress took up the Swadeshi call and the Banaras Session, 1905, presided
over by G.K. Gokhale, supported the Swadeshi and Boycott Movement for Bengal.

9 www.visionias.in ©Vision IAS


o The militant nationalists led by Tilak, Bipin Chandra Pal, Lajpat Rai and Aurobindo Ghosh were,
however, in favour of extending the movement to the rest of India and carrying it beyond the
programme of just Swadeshi and boycott to a full-fledged political mass struggle.
o The aim was now Swaraj and the abrogation of partition had become the ‘pettiest and narrowest of all
political objects” The Moderates, by and large, were not as yet willing to go that far. Hence
statement 2 is not correct.
• The differences between the Moderates and the Extremists, especially regarding the pace of the movement
and the techniques of struggle to be adopted, came to a head in the 1907 Surat session of the Congress
where the party split with serious consequences for the Swadeshi Movement.

Q 27.B
• The subsidiary alliance system was used by Lord Wellesley, who was governor-general from 1798–
to 1805, to build an empire in India. Under the system, the allying Indian state’s ruler was compelled to
accept the permanent stationing of a British force within his territory and to pay a subsidy for its
maintenance.
• Dupleix was the originator of the practice of a subsidiary alliance in India. He placed a French
army at Hyderabad at the expense of the subahdar.
• The Indian ruler had to agree to the posting of a British resident in his court. The Indian ruler could
not employ any European in his service without the prior consultation with the Company. Nor could he go
to war or negotiate with any other Indian ruler without consulting the governor-general. In return for all
this, the British would defend the ruler from his enemies and adopt a policy of non-interference in the
internal matters of the allied state. Hence option (b) is the correct answer.
• One of the objectives behind Wellesley’s strengthening of the subsidiary alliance system was to keep the
French from reviving and expanding their influence in India. Around this time, the fear of
Napoleon’s expedition towards the East was very real for the British who felt that the French could
attack the western coast of India from their colony of Mauritius.
• Wellesley’s policy of subsidiary alliance was, in fact, an extension of the ring-fence system which sought
to reduce the Indian states into a position of dependence on the British government.

Q 28.B
• The British conquered Bengal first and then they reordered its rural society and established a regime of
land rights and revenue system. The first revenue collection system was introduced in Bengal and Bihar. It
was known as the Permanent settlement by Lord Cornwallis.
• As per this system, the zamindars and taluqdars were recognised as estate owners and were given revenue
collection responsibility. They had to collect land revenue from farmers, pass on the share fixed by British
to them and keep the rest. Since the revenue rate were permanent they were kept high. It was argued that
as they couldn't be increased the subsequent loss to British had to be overcome initially. The British felt
this system would benefit British, Zamindars who would invest in the land to improve its productivity as it
would increase their share and farmers too. But the system was a big failure.
• The jotedars were rich farmers who cultivated land directly became more powerful under this
system. Jotedar was a class of rich peasants who were most powerful in North Bengal. They were
known in different areas as haoladars, gantidars, bargadars and mandals. They resisted payment of rent to
zamindars as they were happy to see them in trouble. The jotedars also bought most the land auctioned of
a defaulting zamindar. Jotedars soon possessed thousand acres of land and became powerful in the rural
society. The zamindars couldn't get relief from the judicial process too as it was overburdened. So their
influence decreased.
• Hence option (b) is the correct answer.

Q 29.B
• The government of India which was generally pro-capitalist took some half-hearted measures to mitigate
the sorry state of affairs in modern factories, many of which were owned by Indians.
• The manufacturers of Britain constantly put pressure on the government to pass factory laws. They were
afraid that cheap labour would enable Indian manufacturers to outsell them in the Indian market.
• The first Factory Act was passed in 1881. This primarily dealt with the problem of child labour.
Hence, statement 1 is correct.
o employment of children under 7 years of age prohibited
o working hours restricted to 9 hours per day for children
o children to get four holidays in a month
10 www.visionias.in ©Vision IAS
o hazardous machinery to be properly fenced off
• The Indian Factory Act, 1891
o increased the minimum age (from 7 to 9 years) and the maximum (from 12 to 14 years) for children
o reduced maximum working hours for children to 7 hours a day
o fixed maximum working hours for women at 11 hours per day with a one-and-a-half-hour
interval. the working hours for men were left unregulated. Hence, statement 2 is not correct.
o provided weekly holiday for all.
• But these laws did not apply to British-owned tea and coffee plantations where the labour was
exploited ruthlessly and treated like slaves. The government helped these planters by passing laws such
as those which made it virtually impossible for a labourer to refuse to work once a contract was entered
into. A breach of contract was a criminal offence, with a planter having the right to get the defaulting
labourer arrested. Hence statement 3 is correct.

Q 30.A
• The East India Association was organized by Dadabhai Naoroji in 1866 in London to discuss the
Indian question and influence public men in England to promote Indian welfare. Later, branches of the
association were started in prominent Indian cities. Hence, pair 1 is not correctly matched.
• The Indian League was started in 1875 by Sisir Kumar Ghosh with the object of “stimulating the
sense of nationalism amongst the people” and encouraging political education. Hence, pair 3 is not
correctly matched.
• The Poona Sarvajanik Sabha was a precursor organization to the Indian National Congress. It was
established due to the combined efforts of M.G. Ranade (justice Ranade), S. H. Chiplunkar, Ganesh
Vasudeo Joshi, and other Indian leaders. Hence, pair 2 is correctly matched.
• The Indian Association of Calcutta (also known as the Indian National Association) superseded the
Indian League and was founded in 1876 by younger nationalists of Bengal led by Surendranath
Banerjea and Ananda Mohan Bose, who were getting discontented with the conservative and pro-
landlord policies of the British Indian Association.
• The Indian Association was the most important of pre-congress associations and aimed to “promote by
every legitimate means the political, intellectual and material advancement of the people.”

Q 31.C
• The East India Company, which ruled huge tracts of the subcontinent, recognised that Tipu's powerful
army was one of the greatest threats to their expansion in India. It fought three wars against Tipu and his
father, Hyder Ali, between 1767 and 1792.
• The threat from Mysore was finally removed on 4 May 1799, when the British - supported by the army of
their Indian ally, the Nizam of Hyderabad - stormed and captured Tipu's capital, Seringapatam, after a
month-long siege. Tipu was killed in the fighting, and with his death the Fourth Mysore War (1799)
ended.
• After the war nearly half of Tipu's dominions were divided between the British and their ally, the
Nizam. The reduced Kingdom of Mysore was restored to the descendants of the original rajas from
whom Haidar Ali had seized power. Hence statement 1 is not correct and statement 2 is correct.
• A special treaty of subsidiary alliance was imposed on the new Raja by which the Governor-
General was authorised to take over the administration of the state in case of necessity. Mysore was
in fact made a complete dependency of the company. Hence statement 3 is correct.

Q 32.D
• Recent context: The fifteenth session of the Conference of the Parties (COP-15) of the United Nations
Convention to Combat Desertification (UNCCD) has begun in Abidjan, Côte d’Ivoire. Hence statement
1 is correct.
• COP15 theme, ‘Land. Life. Legacy: From scarcity to prosperity is a call to action to ensure land, the
lifeline on this planet, continues to benefit present and future generations. Hence statement 2 is correct.
• At COP-15, 197 parties will discuss drought, land restoration, and related enablers such as land rights,
gender equality, and youth empowerment.
• In 2019, India hosted COP-14 of UNCCD and committed to restoring 26 million hectares of degraded
land by 2030.
• UNCCD was adopted in 1994 to protect and restore our land and ensure a safer, just, and more sustainable
future.

11 www.visionias.in ©Vision IAS


o It is the sole legally binding international agreement to address desertification and the effects of
drought. Hence statement 3 is correct.
 It addresses arid, semi-arid, and dry sub-humid areas, known as drylands, where some of the most
vulnerable ecosystems and peoples can be found.
o The permanent secretariat is located in Bonn, Germany.

Q 33.C
• Recently, the various reports highlighted the impact on the Dhap Dance of the Kandha tribe of the Kosal
region of Odisha, by the COVID-19 and its related protocol. Hence option (c) is the correct answer.
• The dance is performed both by males and females together during the marriage ceremony and more often
for the sake of recreation. The dance is named so because of the accompanying instrument called ‘Dhap.’
• The damp is in the shape of a Khanjari made up of wood with one side open and the other side covered
with a piece of animal skin.
• Dhap performers use musical instruments like Dhol, Ramtali, Timurli, Tasa, Madhuri, Nishan,
Madal, Ghumra, Ghudka, Gini, and Jhanj apart from the chief and important Dhap instrument.

Q 34.B
• Statement 1 is not correct: The Congress could not attempt a complete overhaul of the agrarian
structure by completely eliminating the zamindari system. This, for two reasons. According to the
constitutional structure of the 1935 Act, the provincial Ministries did not have enough powers to do so.
They also suffered from an extreme lack of financial resources, for the lion’s share of India’s revenues
was appropriated by the Government of India. The Congress Ministries could also not touch the existing
administrative structure, whose sanctity was guarded by the Viceroy’s and Governor’s powers.
• Statement 2 is correct: In U.P. a tenancy act was passed in October 1939 which gave all statutory
tenants both in Agra and Oudh full hereditary rights in their holdings while taking away the
landlord’s right to prevent the growth of occupancy. The rents of hereditary tenants could be changed
only after ten years, while restrictions were placed on the rights of landlords to enhance rents even after
this period. The basic system of landlordism was,not affected. Furthermore, it was, in the main, statutory
and occupancy tenants who benefited. The interests of the sub-tenants of the occupancy tenants were
overlooked. Agricultural labourers were also not affected.
• Statement 3 is correct: There were, however, certain blemishes on the Congress ministerial record in this
respect. In July 1937, Yusuf Meherally, a Socialist leader, was prosecuted by the Madras
Government for making an inflammatory speech in Malabar, though he was soon let off. In October
1937, the Madras Government prosecuted S.S. Batliwala, another Congress Social leader, for making a
seditious speech and sentenced him to six months’ imprisonment.

Q 35.C
• Large parts of East Bengal were engulfed by agrarian unrest during the 1870s and early 1880s. The
unrest was caused by the efforts of the zamindars to enhance rent beyond legal limits and to prevent
the tenants from acquiring occupancy rights under Act X of 1859.
o This they tried to achieve through illegal coercive methods such as forced eviction and seizure of
crops and cattle as well as by dragging the tenants into costly litigation in the courts. Hence,
statement 1 is correct.
• The peasants were no longer in a mood to tolerate such oppression. In May 1873, an agrarian league or
combination was formed in Yusufshahi Parganah in Pabna district to resist the demands of the zamindars.
• The league organized a rent- strike — the ryots were to refuse to pay the enhanced rents — and
challenged the zamindars in the courts. Funds were raised from the ryots to meet the costs. The struggle
gradually spread throughout Pabna and then to the other districts of East Bengal.
• Everywhere agrarian leagues were organized, rents were withheld and zamindars fought in the
courts. The main form of struggle was that of legal resistance. There was very little violence — it
only occurred when the zamindars tried to compel the ryots to submit to their terms by force.
Hence, statement 2 is correct.
• In the course of the movement, the ryots developed a strong awareness of the law and their legal rights
and the ability to combine and form associations for peaceful agitation.
• Though peasant discontent smoldered till 1885, many of the disputes were settled partially under official
pressure and persuasion and partially out of the zamindar‘s fear that the united peasantry would drag them
into prolonged and costly litigation.
o Many peasants were able to acquire occupancy rights and resist enhanced rents.
12 www.visionias.in ©Vision IAS
• The Government rose to the defence of the zamindars wherever violence took place. Peasants were then
arrested on a large sale. But it assumed a position of neutrality as far as legal battles or peaceful agitations
were concerned.
• The Government also promised to undertake legislation to protect the tenants from the worst
aspects of zamindari oppression, a promise it fulfilled however imperfectly in 1885 when the Bengal
Tenancy Act was passed.
• What persuaded the zamindars and the colonial regime to reconcile themselves to the movement was the
fact that its aims were limited to the redressal of the immediate grievances of the peasants and the
enforcement of the existing legal rights and norms.
o It was not aimed at the zamindari system. It also did not have at any stage an anti-colonial
political edge. The agrarian leagues kept within the bounds of law, used the legal machinery to
fight the zamindars, and raised no anti-British demands. The leaders often argued that they were
against zamindars and not the British. Hence, statement 3 is not correct.
• A number of young Indian intellectuals supported the peasants’ cause. These included Bankim Chandra
Chatterjea and R.C. Dutt.
o Later, in the early 1880s, during the discussion of the Bengal Tenancy Bill, the Indian
Association, led by Surendranath Banerjee, Anand Mohan Bose and Dwarkanath Ganguli,
campaigned for the rights of tenants, helped form ryot’ unions, and organized huge meetings of
up to 20,000 peasants in the districts in support of the Rent Bill. Hence, statement 4 is correct.
o The Indian Association and many of the nationalist newspapers went further than the Bill. They asked
for permanent fixation of the tenant’s rent.
Q 36.B
• Since the early 1920s, efforts were being made by various capitalists like G.D. Birla and Purshottamdas
Thakurdas to establish a national level organization of Indian commercial, industrial and financial
interests (as opposed to the already relatively more organized European interests in India) to be able to
effectively lobby with the colonial government. This effort culminated in the formation of the
Federation of Indian Chambers of Commerce and Industry (FICCI) in 1927, with a large and
rapidly increasing representation from all parts of India. The FICCI was soon recognized by the
British government as well as the Indian public in general, as representing the dominant opinion as well as
the overall consensus within the Indian capitalist class. Hence statement 1 is not correct and statement
2 is correct.
• The leaders of the capitalist class also clearly saw the role of the FICCI as being that of ‘national
guardians of trade, commerce and industry,’ performing in the economic sphere in colonial India the
functions of a national government. The FICCI was, however, not to remain merely a sort of trade union
organization of the capitalist class fighting for its own economic demands and those of the nation. The
leaders of the capitalist class now clearly saw the necessity of, and felt strong enough for, the class to
effectively intervene in politics.

Q 37.D
• The idea of the INA was first conceived in Malaya by Mohan Singh, an Indian officer of the British
Indian Army, he went to the Japanese for help to start an army with the prisoners of war. This initiative
received great support from the Japanese Army and from the ethnic Indian population of South-East Asia.
But later disagreements emerged between Mohan Singh and Japanese Army Command regarding
the autonomy of the Indian National Army and the way Indians were treated by the Japanese
soldiers, and later led to the disbandment and the arrest of Mohan Singh. Hence statement 1 is
correct.
• Bose went to Singapore and set up the Provisional Government of Free India on 21 October, 1943. The
Provisional Government then declared war on Britain and the United States, and was recognised by the
Axis powers and their satellites. He set up two INA headquarters, in Rangoon and in Singapore, and
began to reorganize the INA. Recruits were sought from civilians, funds were gathered from
Indian’s all over the world which was managed by a separate bank (Azad Hind Bank). He also set
up 4 Brigades in the army and named them after Gandhi, Nehru, Chandrasekhar Azad and
Himself. Hence statement 2 is correct.
• One INA battalion commanded by Shah Nawaz was allowed to accompany the Japanese Army to the
Indo-Burma front and participate in the Imphal campaign. But the discriminatory treatment which
included being denied rations, arms and being made to do menial work for the Japanese units, completely
demoralized the INA men. The failure of the Imphal campaign, and the steady Japanese retreat thereafter,
quashed any hopes of the INA liberating the nation. Hence statement 3 is correct.

13 www.visionias.in ©Vision IAS


Q 38.C
• During World War II several economic plans were devised for future Independent India, the most
important being the Bombay Plan of 1944-45 drawn up by the big three of the Indian capitalist
world -— J.R.D. Tata, G.D. Birla and Sri Ram. This plan visualized far-reaching land reforms, a large
public sector and massive public and private investment. Hence, statement 1 is not correct.
• The signatories of the plan were Purshottamdas Thakurdas, J.R.D. Tata, G.D. Birla, Ardeshir
Dalal, Sri Ram, Kasturbhai Lalbhai, A.D. Shroff and John Mathai. Hence, statement 2 is correct.
• The Bombay Plan seriously took up the question of rapid economic growth and equitable
distribution, even arguing for the necessity of partial nationalization, the public sector, land reform
and a series of workers’ welfare schemes. Hence, statement 3 is correct.
• The basic assumption made by the Bombay planners was that the plan could be implemented only by an
independent national Government.

Q 39.D
• Swami Dayananda Saraswati (1824-83) founded the Arya Samaj in 1875. He was a Sanskrit expert who
had never studied English. He issued the slogan, "Back to the Vedas."
• Believes in the infallibility of the Vedas and regards them as the ultimate source of all truth and
knowledge. Hence statement 3 is correct.
• It was believed that post-Vedic books such as Puranas were to blame for the contamination of the Vedic
religion.
• Opposes God's idolatry and reincarnation idea, but supports the notion of 'Karma' and soul
transmigration. Hence statement 1 is correct.
• Dayanand also rejected the doctrine of fate/destiny Niyati.
• Believes in a single God who does not have a physical existence.
• Rejects Brahmanical domination over Hindu spiritual and social life. Brahmins' claim to be
conduits between man and God is condemned.
• Supported the Four Varna System, however, it should be based on merit rather than birth.
• Study of western Sciences for the development of the young generation was strongly supported.
Hence statement 2 is correct.
• Everyone has an equal position in the spiritual and social lives of Hindus.
• Advocated for women's equality in society. There is no place for any type of gender discrimination
against women.
• Supported widow remarriage and female education while opposing polygamy, child marriage, Sati, and
other practices.
• After the death of Dayananda in 1883, the work of the samaj was carried on by illustrious members.
Education was an all-important field for the samaj.
• The Dayananda AngloVedic (D.A.V.) College was established in 1886 in Lahore. But a difference of
opinion between two groups in the samaj arose over the curriculum of the D.A.V. College.
• One group was known as the College Party (some sources say ‘Culture’ Party), among whose leaders
were Lala Hansraj, Lala Lal Chand, and Lala Lajpat Rai, and the other was the Mahatma (later Gurukul)
Party led by Guru Datta Vidyarthi and Lala Munshi Ram (who later came to be known as Swami
Shraddhanand).
• While the College Party favored the government curriculum and English education to meet
economic and professional needs.
• Mahatma Party was interested in introducing the study of Sanskrit and Vedic philosophy in the
tradition of ancient gurukuls.
• Later, the issue of vegetarianism also became a point of contention: the College Party had nothing
against non-vegetarianism, claiming that diet was a personal choice and it was not mentioned in the
principles of the samaj; the Mahatma Party was in favor of all the Aryas being strict vegetarians.
• In the end, the Arya Samaj split in 1893 over these issues.

Q 40.D
• In 1920 the British imposed a harsh treaty on the Turkish Sultan or Khalifa. People were furious about
this as they had been about the Jallianwala massacre. Also, Indian Muslims were keen that the Khalifa be
allowed to retain control over Muslim sacred places in the erstwhile Ottoman Empire.
• The Indian Muslims were incensed when they discovered that their loyalty had been purchased during the
War by assurances of generous treatment of Turkey after the War - a promise British statesman had no

14 www.visionias.in ©Vision IAS


intention of fulfilling. The Muslims regarded the Caliph of Turkey as their spiritual head and were
naturally upset when they found that he would retain no control over the holy places in the erstwhile
Ottoman Empire, the jazirat-ul-Arab (Arabia, Syria, Iraq and Palestine). The leaders of the Khilafat
agitation, Mohammad Ali and Shaukat Ali, now wished to initiate a full-fledged movement. Hence
statements 1 and 2 are correct.
• The Congress supported the movement and Mahatma Gandhi sought to conjoin it to the Non -
Cooperation movement. Gandhiji, who had been in close touch with the Khilafat leaders for quite some
time, and was a special invitee to the Khilafat Conference in November 1919, had all along been very
sympathetic to their cause, especially because he felt the British had committed a breach of faith by
making promises that they had no intention of keeping. In February 1920, he suggested to the Khilafat
Committee that it adopt a programme of non-violent non-cooperation to protest the Government’s
behaviour. On 9 June 1920, the Khilafat Committee at Allahabad unanimously accepted the
suggestion of non-cooperation and asked Gandhiji to lead the movement. The movement was launched
formally on 1 August 1920. Hence statement 3 is correct.

Q 41.B
• The third Congress session was held in 1887 at Madras. It was presided by Badruddin Tyabji.
Badruddin Tyabji was born in Bombay on 10 October 1844. After passing the London matriculation he
joined the Middle Temple, became a Barrister in April 1867 - the first Indian Barrister in Bombay - and
rose rapidly in the profession.
o In 1885 he helped to found the Bombay Presidency Association and virtually ran it all by himself.
Soon afterwards, the Indian National Congress held its first session in Bombay under its auspices; and
Tyabji and Qamruddin Tyabji (his brother) were among its delegates.
o He was unanimously elected as the President of its third Session in Madras (1887). He was also
the first Muslim president of Congress. Hence pair 1 is correctly matched.
o Qamruddin Tyabji and he were principally responsible for establishing the Anjuman - I - Islam in
Bombay (1876) "for the betterment and uplift of Mussalmans in every direction".
• The 39th session of Congress was held in 1924 at Belgaum. It was the only session presided by
Mahatma Gandhi. Hence pair 2 is correctly matched.
o Gandhiji brought the strife between the Swarajists and no-changers to an end, by signing a joint
statement with C.R Das and Motilal that the Swarajist Party would carry on work in the legislatures
on behalf of the Congress and as an integral part of the Congress. This decision was endorsed in
December at the Belgaum session of the Congress
• The 51st session of Congress was held in 1938 at Haripura. It was presided by Subhas Chandra
Bose. Hence pair 3 is not correctly matched.
o It was in this session that Congress, then under the presidentship of Subhas Chandra Bose, set up the
National Planning Committee under the chairmanship of Nehru, to draw up a development plan for
free India.

Q 42.A
• The capitalists such as G.L. Mehta who was the president of FICCI, argued in 1943, that ‘A
consistent programme of reforms’ was the most effective remedy against social upheavals.’ It was
with this reform perspective that the ‘Post War Economic Development Committee,’ set up by the
capitalists in 1942, which eventually drafted the Bombay Plan, was to function.
• Its attempt was to incorporate ‘whatever is sound and feasible in the socialist movement’ and see ‘how far
socialist demands could be accommodated without capitalism surrendering any of its essential features.’
• The Bombay Plan, therefore, seriously took up the question of rapid economic growth and equitable
distribution, even arguing for the necessity of partial nationalization, the public sector, land reform and a
series of workers’ welfare schemes.
• Hence, option (a) is the correct answer.

Q 43.C
• Recently, India-UAE Comprehensive Economic Partnership Agreement enters into force that is likely to
benefit about $26 billion worth of Indian products that are currently subjected to 5% import duty by the
UAE
o A Comprehensive Economic Partnership Agreement (CEPA) is one of the types of trade agreement
that consist of a bilateral agreement covering trade in goods and services, investment, competition,
and intellectual property rights. Hence statement 1 is correct.

15 www.visionias.in ©Vision IAS


 Other Trade agreements are - Free Trade Agreement (FTA), Preferential Trade Agreement
(PTA,) and Comprehensive Economic Cooperation Agreement (CECA).
▪ PTA is unilateral trade preferences, including the Generalized System of Preferences
schemes under which developed countries grant preferential tariffs to imports from
developing countries.
▪ FTA is between two or more countries and reduces barriers to imports and exports among
them. This is provided through preferential trade terms, tariff concessions, etc
▪ A CEPA is a more comprehensive trade agreement than a CECA as Comprehensive
Economic Cooperation Agreement generally covers negotiation on trade tariffs and
tariff-rate quotas only. CEPA, on the other hand, also covers the trade in services and
investment and other areas of economic partnership. Hence statement 2 is correct.
• Also, for the first time in any Trade Agreement, a separate Annex on Pharmaceuticals has been
incorporated to facilitate access to Indian pharmaceuticals products, especially automatic registration
and marketing authorization in 90 days for products approved by developed country regulators namely the
United States, the United Kingdom the European Union, and Japan.

Q 44.D
• Recently, the Registrar General of India released the Civil Registration System (CRS) report for
2020. Hence statement 3 is correct.
• The report provides an overview of the working of the CRS in the country and presents a compilation of
data on registered births, deaths, infant deaths, and stillbirths with rural-urban & sex-wise break-
up and sex ratio at birth based on Civil Registration Records.
• CRS is the only source for providing vital rates at the district level regularly. The Population Census
is a decennial exercise and the Sample Registration System is an annual exercise.
• The Registrar General, India (RGI) at the Central Government level coordinates and unifies the
activities of registration throughout the country. However, implementation of the statute is vested
with the state governments.
• Registration of birth and death is done under a central Act “Registration of Birth and Death Act (RBD),
1969”.
o With the enactment of the Act, registration of births, deaths, and stillbirths has become mandatory in
India.
• Births and deaths are registered only at the place of their occurrence and are to be reported within 21
days of the occurrence of the event.
• Findings
o Death registrations increased by 4.75 lakh in the year 2020 as compared to 2019 with total
registered deaths at 81.16 lakh.
o Registered 2.42 crores births in 2020, decreased from 2.48 crores in 2019, with 52% Male and
48% Female.
o Ladakh recorded the highest sex ratio at birth across India, followed by Arunachal Pradesh and
Andaman and Nicobar. Hence statement 1 is correct.
 The Sex Ratio (Number of females per thousand males) at the birth of registered events is an
important indicator to map the sex differential of the population at the beginning of their life.
o None of the States or Union Territories have recorded a sex ratio at birth below 880. Hence
statement 2 is correct.

Q 45.C
• Recently, the Breed-Wise Report of Livestock and Poultry based on the 20th Livestock Census was
released by the Ministry of Fisheries, Animal Husbandry, and Dairying.
• Livestock, as well as poultry birds, were counted according to their breeds as recognized by the National
Bureau of Animal Genetic Resources (NBAGR).
o For the first time breed-wise data was collected by using digital modes instead of paper mode
which is indeed a unique attempt.
• Key highlights of the report –
o The Exotic and Crossbred animals contribute nearly 26.5% of the total cattle population whereas
73.5% are Indigenous and Non-descript cattle.
 The census defines “Exotic” cattle as “the animals which have their origin in other countries.
o The total number of indigenous cattle declined by 6% to 14.21 crore in 2019 from 15.12 crore in
2012. Hence statement 1 is correct.
16 www.visionias.in ©Vision IAS
o The number of Exotic/Crossbred cattle grew from 3.9 crores in 2012 to 5 crores in 2019.
o Crossbred Jersey cow has the highest share in total Exotic/Crossbred cattle and Gir, Lakshmi,
and Sahiwal breeds have the major contribution in total Indigenous cattle. Hence statement 2 is
correct.
o In Sheep, there are 3 exotics and 26 indigenous breeds were found in the country. Among the pure
exotic breeds, Corriedale breed majorly contributes with 17.3% and in the indigenous breeds
the Nellore breed contributes the highest in the category with 20.0% share.
o In Goats, there are 28 indigenous breeds found in the country. The Black Bengal breed contributes
the highest with 18.6%.
o In exotic/crossbred pigs, crossbred pig contributes 86.6% whereas Yorkshire contributes majorly
with 8.4%. In indigenous pigs, the Doom breed major contributes with 3.9%.
o In Horse & Ponies, the share of Marwari breed majorly contributes with 9.8%.In Donkeys, the share
of Spiti breed contributes with 8.3%.
o In Camel, Bikaneri breed majorly contributes with 29.6%.In Poultry, desi fowl, Aseel breed majorly
contribute in both backyard poultry and commercial poultry farm.

Q 46.C
• Calcutta being the first capital of British India, was thick in the process of development. After the Battle
of Plassey with the laying out of Maidan around Fort William, the British entrenched themselves with the
local administration.
• Having been elevated from merchants to rulers, their immediate concern was to provide wide roads for
easy movement of their army, the establishment of commercial interests wherever possible and attend to
the urgent needs of fire control and prevention of epidemics in their seat of power i.e. Calcutta. With
these goals in mind, Governor-General Lord Wellesley (1798-1805) began the planning process with
his prescriptive ‘Minute on Calcutta' in 1803 which led to the setting up of the Lottery Committee
in 1817 — so-called because funds for city development were raised through public lotteries.
• The Lottery Committee (1817) assisted the government in the task of town planning in Calcutta.
The committee was named because funds for town improvement were raised through public
lotteries. Hence option (c) is the correct answer.
• The Lottery Committee commissioned a new city map to provide a comprehensive picture of Calcutta.
Among the main activities of the Committee was road building in the Indian part of the city and clearing
the river bank from encroachments. In its drive to clean up the Indian areas of Calcutta, the committee
removed many huts and displaced the laboring poor, who had now been pushed to the outskirts of
Calcutta.

Q 47.C
• Recent Context: The surviving soldiers of Operation Dudhi were recently felicitated by Assam
Rifles. In 1991, 72 militants were eliminated by the Assam Rifles in a single counter-insurgency
operation that was conducted in Jammu & Kashmir. This remains the most successful counter-
insurgency operation that has been conducted by any security force to date wherein 15 personnel from the
Assam Rifles gunned down 72 militants. Hence option (c) is the correct answer.
• About Assam Rifles
o The Assam Rifles is the oldest branch of the Indian Army, dating back to 1835 under the British Raj,
raised under the name Cachar Levy. The present name of "Assam Rifles" has been used since 1917.
o Meghalaya’s capital Shillong is the headquarters of Assam Rifles, whose operational control is with
the Defence Ministry and administrative control with the Home Ministry.

Q 48.C
• As an answer to the challenge of the Simon Commission, Indian political leaders organized several all-
India conferences to settle communal issues and draw up an agreed constitution for India. The Congress
proposals came in the form of the Nehru Report (chaired by Motilal Nehru) drafted by an all-parties
committee. The Report was put up for approval before an All-Party Convention at Calcutta at the end of
December 1928.
• Nehru report made the following recommendations:
o The Nehru Report recommended that India should be a federation on the basis of linguistic
provinces and provincial autonomy. Hence statement 1 is correct.
o It suggested that seats in central and provincial legislatures be reserved for religious minorities in
proportion to their population. Hence statement 2 is correct.

17 www.visionias.in ©Vision IAS


o The Report recommended the separation of Sind from Bombay and constitutional reform in the
North - West Frontier Province. Hence statement 3 is not correct.
o The elections were be held on the basis of joint electorates . However a large section of the League
led by Mohammed Shafi and the Aga Khan and many other Muslim communal groups were not
willing to give up separate electorates. Hence statement 4 is correct.
o Dominion status on the lines of self-governing dominions as the form of government desired by
Indians.
o Nineteen fundamental rights including equal rights for women, right to form unions, and universal
adult sufferage.
o Full protection to cultural and religious interests of Muslims.
o Complete dissociation of state from religion.

Q 49.A
• Individual Satyagraha was direct result of August Offer, which was rejected by both INC and the
Muslim League. Since Gandhiji thought that the time was not ripe for an another mass struggle and not to
hamper the war efforts of British. He decided start Individual Satyagraha, to convey people’s
disinterest in the war and their lack of distinction between Nazism and the double autocracy that
controlled India. Hence statement 1 is correct.
• Individual Satyagraha was offered by Vinobha Bhave first and followed by Jawaharlal Nehru in
1941, this forced the British to come up with Cripps Proposal which was different from August offer and
it provided way for Constituent Assembly in future. Hence statement 2 is not correct.

Q 50.D
• The main form of political work by the early Communists was to organize Workers ’ and Peasants’
Parties (WPPs) and work through them . The first such organization was the Labour-Swaraj Party of
the Indian National Congress organized by Muzaffar Ahmed, Qazi Nazrul Islam, Hemanta Kumar
Sarkar, and others in Bengal in November 1925 . In late 1926, a Congress Labour Party was formed in
Bombay and a Kirti-Kisan Party in Punjab. A Labour Kisan Party of Hindustan had been functioning in
Madras since 1923.
• During the 1920s, WPPs sprang up in Bengal, Bombay, Punjab, UP and Ajmer- Marwara with basically
the same orientation and programme. The basic objective of the WPPs was to work within the Congress
to give it a more radical orientation and make it The party of the people 5 and independently organize
workers and peasants in class organizations, to enable first the achievement of complete independence
and ultimately of socialism .
• It was in the second half of the 1920s that the consolidation of various Left ideological trends occurred and
began to have a significant impact on the national movement. Various Communist groups in different
parts of India had by early 1927 organized themselves into the Workers5 and Peasants5 Parties (WPP),
under the leadership of people like S.A. Dange, Muzaffar Ahmed , P.C. Joshi and Sohan Singh Josh.
Hence option (d ) is the correct answer.
• The WPPs, functioning as a left-wing within the Congress, rapidly gained in strength within the Congress
organization at the provincial and the all India levels. Also, by working within a broad Left from under
the WPPs, Communist influence in the trade union movement, marginal till early 1927, had become very
strong indeed , by the end of 1928.

Q 51.B
• The Securities and Exchange Board of India (SEBI) has relaxed norms for anchor investors by keeping
the 30 days lock- in period till June 30 for issues exceeding ? 10,000 crore.
o The Life Insurance Corporation (LIC) of India has raised over ? 5 ,600 crores from 123 anchor
investors ahead of its much-awaited initial public offering (IPO).
• Anchor Investors, also known as cornerstone investors, are qualified institutional buyers who buy
shares in large quantities such as mutual funds, foreign institutional investors, banks, provident funds,
etc. They invest in shares before it is made available for IPO, to make it more attractive to invest by
the public.
o They have guaranteed allotment of shares ahead of company IPO, up to 60% of the qualified
institutional investors 5 pool, to help the company in bringing other investors.
• Hence option ( b) is the correct answer.

18 ©Vision IAS
Q 52.C
• Universities Act, 1904 was the outcome of deliberations held at the Educational Conference at Simla
in 1901 and the recommendations put forward by the Universities Commission of 1902. It came into
force on 1 September 1904. Indian Universities Act 1904 ensured greater government control over
universities, which it described as factories producing political revolutionaries.
• The National Council of Education- Bengal (or NCE - Bengal) was an organization founded by Satish
Chandra Mukherjee and other Indian nationalists with Aurobindo Ghose as principal in Bengal in 1906
to promote science and technology as part of a swadeshi industrialization movement.
• When Lord Hardinge assumed charge as Governor-General of India. On the occasion of the visiting His
Majesty George V and holding of Darbar at Delhi on 12th December 1911, the partition of Bengal was
cancelled.
• Hence option (c) is the correct answer.

Q 53.C
• Recent context: The United Kingdom (UK) has highlighted the need to renegotiate the Northern
Ireland Protocol. According to the UK, the protocol has the potential to create so many problems
and hence suggested rewriting it or even abandoning it in the absence of renegotiations.
• Special arrangements were needed for Northern Ireland after the UK voted for Brexit in 2016. This is
because it is the only part of the UK with a land border with an EU country - the Republic of Ireland.
Before Brexit, it was easy to transport goods across this border because both sides had the same EU trade
rules. No checks or paperwork were necessary.
• After Brexit, a new system was needed because the EU has strict food rules and requires border checks
when certain goods, such as milk and eggs, arrive from non-EU countries. The border is also a sensitive
issue because of Northern Ireland's troubled political history. It was feared that cameras or border posts
could become a target and lead to instability. The UK and the EU agreed that protecting the 1998
Northern Ireland peace deal - the Good Friday Agreement - was an absolute priority. So, they
agreed to the Northern Ireland Protocol as part of the Brexit withdrawal agreement, which is now
part of international law.
• Northern Ireland Protocol aims to resolve the issue of the border between Northern Ireland and the
Republic of Ireland. Northern Ireland is part of the UK and the Republic of Ireland remains part of the
European Union (EU).
• Under the protocol, instead of checking goods at the Irish border, the protocol agreed that any
inspections and document checks would be conducted between Northern Ireland and Great Britain
(England, Scotland, and Wales). These take place at Northern Ireland's ports. It was also agreed
Northern Ireland would keep following EU rules on product standards.
• Hence option (c) is the correct answer

19 www.visionias.in ©Vision IAS


Q 54.D
• The Revolt of 1857 was a major uprising in India in 1857–58 against the rule of the British East India
Company.
• It began on 10 May 1857 in the form of a mutiny of sepoys of the Company's army in the garrison town of
Meerut.
• It was fed by resentments born of diverse perceptions, including invasive British-style social reforms,
harsh land taxes, summary treatment of some rich landowners and princes, as well as skepticism about the
improvements brought about by British rule.
• Causes of The Revolt:
o Political and Administrative Causes:
 British policy of expansion through the Doctrine of Lapse and direct annexation.
 A large number of Indian rulers and chiefs were dethroned which raised suspicion and fuelled
resentment in the minds of ruling families. For egRani Lakshmi Bai’s adopted son was not
permitted to sit on the throne of Jhansi. Satara, Nagpur, and Jhansi were annexed under the
Doctrine of Lapse. Awadh was annexed on the pretext of maladministration.
 Common people were hard hit by the prevalence of corruption at the lower levels of
administration.
 The middle and upper classes of Indian society, particularly in the North were hard hit by their
exclusion from the well-paid higher posts in the administration.
o Social and Religious Cause:
 Western Civilisation was gradually spreading all over the country.
 Hindu law of inheritance enables Christian converts to inherit their ancestral properties.
 The abolition of practices like sati and female infanticide, and the legislation legalizing widow
remarriage, was believed as threats to the established social structure.
 Western education system posed a threat to the orthodox section of Hindu as well as Muslim
communities.
o Economic Cause:
 Peasants and zamindars were explored through the imposition of heavy taxes on land and harsh
consequences in case of failure to do so including confiscation of landholdings.
 The influx of British manufactured goods into India ruined local industries, particularly the textile
industry of India.
o Military Causes:
 Indian sepoys were paid less than a European sepoy of the same rank.
 They were required to serve in areas far away from their homes.
 Sepoys would not be given the foreign service allowance (batta) when serving in Sind or in
Punjab.
 General Services Enlistment Act required the sepoys must be ready to serve even in British
land across the sea.
 Military authorities forbade the sepoys to wear caste and sectarian marks.
 Use of greased cartridges which were rumored to contain animal fat hurt the religious sentiment
of Indian Sepoys.
• Hence option (d) is the correct answer.

Q 55.B
• Valentine Chirol of London Times published articles in 1910 in a book called ‘The Indian unrest’ in
which he called Bal Gangadhar Tilak the “Father of Indian unrest”. Tilak decided to go to England to
pursue the libel case that he had filed against Valentine Chirol. Hence pair 1 is correctly matched.
• Young India was a weekly paper or journal in English started by Mahatma Gandhi. Through this
work, he desired to popularise India's demand for self-government or Swaraj. Hence pair 2 is correctly
matched.
• Tarak Nath Das, an Indian student, and one of the first leaders of the Indian community in North
America to start a paper (called Free Hindustan) realized that while the British government was keen
on Indians going to Fiji to work as laborers for British planters, it did not want them to go to North
America where they might be infected by ideas of liberty. Hence pair 3 is not correctly matched.
• Bal Gangadhar Tilak, was the outstanding leader of militant nationalism. Born in 1856, Tilak
devoted his entire life to the service of his country. In 1881, along with G.G. Agarkar, he founded
the newspaper Kesari (in Marathi) and Mahratta (in English). In 1888, he took over the two papers

20 www.visionias.in ©Vision IAS


and used their columns to spread discontent against British rule and to preach national resistance to
it. Hence pair 4 is not correctly matched.

Q 56.C
• Madras developed by incorporating innumerable surrounding villages and creating opportunities and
spaces for a variety of communities. Several different communities came and settled in Madras,
performing a range of economic functions.
• The dubashes were Indians who speak two languages-the local languages and English. They worked
as agents and merchants, acting as intermediaries between Indian society and the British. They used
their privileged position in the government to acquire wealth.
• Initially Vellalars, a rural caste monopolized jobs for Company by taking advantage of new
opportunities provided by British rule. After the spread of English education Bramhins started
competing with them for similar positions in the administration.
• Telugu Komatis were a powerful commercial group that controlled the grain trade in Madras city.
Gujratis performs banking operations in the city since the eighteenth century.
• Paraiyars and Vanniyars formed the labouring poor.
• The Nawab of Arcot settled in nearby Triplicane which became the nucleus of substantial Muslim
settlement. Hence option (c) is the correct answer.

Q 57.B
• The Cripps mission was an attempt in late March 1942 by the British government to secure full
Indian cooperation and support for their efforts in World War II. The mission was headed by Sir
Stafford Cripps, a senior left-wing politician and government minister in the War Cabinet, who had
actively supported the Indian national movement. Hence statement 1 is correct.
• The main proposals of the mission were as follows:
o An Indian Union with a dominion status would be set up; it would be free to decide its relations
with the Commonwealth and free to participate in the United Nations and other international
bodies. Hence statement 2 is not correct.
o After the end of the war, a Constituent Assembly would be convened to frame a new
constitution. Members of this assembly would be partly elected by the provincial assemblies through
proportional representation and partly nominated by the princes. Hence statement 3 is correct.
o The British government would accept the new constitution subject to two conditions:
 any province not willing to join the Union could have a separate constitution and form a separate
Union, and
 the new constitution- making body and the British government would negotiate a treaty to effect
the transfer of power and to safeguard racial and religious minorities.
o In the meantime, defence of India would remain in British hands and the governor-general’s powers
would remain intact.

Q 58.B
• The historic Dandi March, which marked the launch of the Civil Disobedience Movement, began on
March 12 of 1930. Mahatma Gandhi broke the salt law by picking up a lump of salt at Dandi on April 6.
The violation of the law was seen as a symbol of the Indian people’s resolve not to live under British-
made laws and therefore under British rule.
• A brief survey of the nature of the Civil Disobedience Movement in different parts of the subcontinent
is given below:
• In Bengal, the onset of the monsoon, which made it difficult to make salt, brought about a shift to anti-
chowkidar and anti-Union Board agitation. Here too, villagers withstood severe repression, losing
thousands of rupees worth of property through confiscation and destruction, and having to hide for days in
forests to escape the wrath of the police.
• In Assam, a powerful agitation led by students was launched against the infamous ‘Cunningham
Circular’ which forced students and their guardians to furnish assurances of good behavior. Hence
option (b) is the correct answer.
• Defiance of forest laws assumed a mass character in Maharashtra, Karnataka, and the Central
Provinces, especially in areas with large tribal populations that had been the most seriously affected by
the colonial Government’s restrictions on the use of the forest.

21 www.visionias.in ©Vision IAS


Q 59.D
• The Indian National Congress was not the first political organisation in India. However, most of the
political associations in the early half of the nineteenth century were dominated by wealthy and
aristocratic elements. They were local or regional in character.
• The associations organised various campaigns before the Indian National Congress appeared on the scene.
Nationalist Indian demands during Pre-Congress Campaigns were —
o for imposition of import duties on textile import; hence option 1 is not correct.
o no expansion in Afghanistan or Burma; hence option 2 is correct.
o the right to bear arms, freedom of the Press;
o reduction of military expenditure;
o higher expenditure on famine relief; hence option 3 is correct.
o Indianization of the civil services;
o the right of Indians to join the semi-military volunteer corps; hence option 4 is correct.
o the right of Indian judges to try Europeans in criminal cases; hence option 5 is correct.
• But these were demands that a colonial regime could not easily concede, for that would undermine its
hegemony over the colonial people.

Q 60.D
• Permanent Settlement system: It was introduced in Bengal, Bihar, Orissa, Banaras division of modern
UP, and Northern Carnatic in the 18th century. The zamindars were recognized as the owners of land and
ten years of settlement was made with them in 1790. In 1793, under Governor-General Lord Cornwallis
the decennial settlement was declared permanent and the zamindars and their legitimate successors were
allowed to hold their estates.
• Features
o Zamindars were not only to act as agents of the government but to become the owners of the entire
land.
o Zamindar's right of ownership was made hereditary and transferable.
o Cultivators were reduced to the status of mere tenants.
o Zamindars were to pay 10/11th of the rent they derived from peasants and this was fixed in
perpetuity.
• Zamindars had to pay the revenue on the due date even if the crops failed. Moreover, the initial
fixation of the land revenue under this system was made arbitrarily and without any consultation
with zamindars.
• Consequences:
o The state proved to be a great loser in the long run as a prospective share in the increase in land
revenue was sacrificed.
o Since the land revenue was to be fixed for perpetuity, it was fixed at a high level – the absolute
maximum, and the customary rates were increased. This placed a high burden on revenue.
o Though a fixed revenue demand was placed with the Zamindars, no rules were placed regulating the
collection of revenue from the peasants. As a result, Zamindars placed exorbitant demands.
o Absentee landlordism was a consequential feature. Zamindars took no interest in the development of
agriculture.
o Peasants suffered from the double injustice of surrendering their property rights and being left entirely
at the mercy of Zamindars
• Hence, all three statements are correct.

Q 61.D
• Ahrar Movement was founded in 1910 under the leadership of Maulana Muhammad Ali, Hakim Ajmal
Khan, Hasan Imam, Maulana Zafar Ali Khar and Mazhar-ul-haq. Hence statement 2 is correct.
• The movement supported the idea of self-government and advocated active paticipation in militant
nationalist movement. Hence statement 1 is correct.
• The movement was against loyalist politics of Aligarh school and zamindars. Hence statement 3 is
correct.
• On the political front, the Deoband school welcomed the formation of the Indian National Congress.
• In 1888 it issued a fatwa (religious decree) against Syed Ahmed Khan’s organisations, the United Patriotic
Association and the Mohammaden Anglo-Oriental Association.
• Some critics attribute Deoband’s support to the nationalists more to its determined opposition to Syed
Ahmed Khan than to any positive political philosophy.

22 www.visionias.in ©Vision IAS


Q 62.B
• The All India Women’s Conference (AIWC), founded by Margaret Cousins in 1927, was perhaps
the first women’s organization with an egalitarian approach. Hence statement 1 is not correct.
• Its first conference was held at Ferguson College, Pune. Important founding members included Maharani
Chimnabai Gaekwad, Rani Sahiba of Sangli, Sarojini Naidu, Kamla Devi Chattopadhyaya, and Lady
Dorab Tata.
• Its objectives were to work for a society based on principles of social justice, integrity, equal rights,
and opportunities; and to secure for every human being, the essentials of life, not determined by the
accident of birth or sex but by planned social distribution. Hence statement 2 is correct.
• For this purpose, the AIWC worked towards various legislative reforms before and after India’s
independence, some examples being Sarda Act (1929), Hindu Women’s Right to Property Act (1937),
Factory Act (1947), etc.

Q 63.D
• Jyotiba Phule (1827-1890), born in Satara, Maharashtra, belonged to the mali (gardener)
community and organized a powerful movement against upper caste domination and brahminical
supremacy.
• Phule founded the Satyashodhak Samaj (Truth Seekers’ Society) in 1873, with the leadership of the
samaj coming from the backward classes, malis, telis, kunbis, saris, and dhangars.
• In 1851, he started a girl's school at Pune with his wife and soon many other schools came up.
• The main aims of the movement were (i) social service, and (ii) the spread of education among women
and lower caste people. Phule’s works, Sarvajanik Satyadharma and Gulamgiri, became sources of
inspiration for the common masses.
• Phule used the symbol of Rajah Bali as opposed to the brahmins’ symbol of Rama. Phule aimed at
the complete abolition of the caste system and socio-economic inequalities; he was against Sanskritic
Hinduism.
• This movement gave a sense of identity to the depressed communities as a class against those brahmins
who used religion and the blind faith of the masses to exploit the masses for personal monetary gain.
• Hence option (d) is the correct answer.

Q 64.D
• After almost three decades, the auspicious Navgrah Ashtamangalam Pooja was held recently, at the
ancient Martand Sun temple in Mattan (Jammu & Kashmir).
o Martand Temple was built by King Lalitaditya of the Karkota Dynasty of Kashmir (724 to 761
CE) in honor of Sun or Martand in the 8th century. Hence statement 1 is correct.
 It is one of the oldest sun temples such as Konark, Almora, Modhera, and Borsad Sun Temple.
o It is considered an example of Kashmiri architectural skills built in a square field using limestone
and pillars in Greek Pattern. Hence statement 2 is correct.
• In the 15th century, it was destructed on the order of Muslim ruler Sikander Butshikan.
• It has been tagged as a site of national importance and protected by the Archaeological Survey of
India. Hence statement 3 is correct.

Q 65.C
• An English association or company to trade with the East was formed in 1599 under the auspices of a
group of merchants known as the Merchant Adventurers. The company was granted a Royal Charter
and the exclusive privilege to trade in the East by Queen Elizabeth on 31 December 1600 and was
popularly known as the East India Company (EIC). From the beginning, it was linked with the
monarchy: Queen Elizabeth was one of the shareholders of the company. Hence statement 1 is correct.
• In 1608 EIC decided to open a factory at Surat on the West coast of India and sent Captain Hawkins to
Jahangir’s Court to obtain Royal favours. In 1613, Jahangir issued a farman permitting the English to
establish a factory permanently at Surat. Therefore, EIC's first permanent factory was set up in
Surat. Hence statement 2 is not correct and statement 3 is correct.
• From 1600 to 1757 the East India Company’s role in India was that of a trading corporation which
brought goods or precious metals into India and exchanged them for Indian goods like textiles, spices,
etc., which it sold abroad. Its profits came primarily from the sale of Indian goods abroad. Naturally, it
tried constantly to open new markets for Indian goods in Britain and other countries. Thereby, it increased
the export of Indian manufacturers and thus encouraged their production.

23 www.visionias.in ©Vision IAS


Q 66.B
• Statement 1 is not correct: Lokamanya Tilak, who had developed a close association with Bombay
work, was one of the moving spirits in the formation of the AITUC. In the first conference, Lala Lajpat
Rai was elected as the President and V.M. Pawar was the first General Secretary.
S.No. Dates of Session Place President General Secretary
First Session October, 1920 Bombay Lala Lajpat Rai V.M.Pawar
Second Session 30 November-2 December, 1921 Jhari Joseph Baptista Diwan Chamanlal
Thrid Session 24-26 March, 1923 Lahore C.R. Das Diwan Chamanlal
Seventh Session 12-13 March, 1927 Delhi Diwan Chamanlal N.M. Joshi
Ninth Session 18-20 December, 1928 Jharia Jawaharlal Nehru N.M. Joshi
Subhash Chandra
Tenth Session 28 November - 1 December, 1929 Nagpur S.V. Deshpande
Bose
• The manifesto issued to the workers by the AITUC urged them not only to organize themselves but also to
intervene in nationalist politics: ‘Workers of India! . . . Your nation’s leaders ask for Swaraj, you must not
let them, leave you out of the reckoning. Political freedom to you is of no worth without economic
freedom. You cannot therefore afford to neglect the movement for national freedom. You are part and
parcel of that movement. Lajpat Rai was among the first in India to link capitalism with imperialism and
emphasize the crucial of the working class in fighting this combination.
• Statements 2 and 3 are correct: The Indian National Congress at its Gaya session in 1922 welcomed
the formation of the AITUC and formed a committee consisting of prominent Congressmen to assist
its work. Apart from Lajpat Rai, several of the leading nationalists of the time became closely associated
with the AITUC. C.R. Das presided over its third and fourth sessions, and among the other prominent
names were the of C.F. Andrews, J.M. Sengupta, Subhas Bose, Jawaharlal Nehru, and Satyamurti.

Q 67.D
• At the first post-War Congress session in September 1945 at Bombay, a strong resolution was adopted
declaring Congress support for the Indian National Army (INA) cause. The defence of the INA
prisoners was taken up by the Congress and Bhulabhai Desai, Tej Bahadur Sapru, K.N. Katju,
Jawaharlal Nehru and Asaf Ali appeared in court at the historic Red Fort trials. Hence option (d) is the
correct answer.
• The three INA generals arraigned for the first trial were P. K. Sehgal, Shah Nawaz Khan, Gurbaksh
Singh Dhillon. The defense lost the case and the defendants were declared guilty, but the British sensing
the popular mood found that there could be an uprising throughout the country and was forced to
commute the sentences of the convicted trio and release them.

Q 68.B
• On Gokhale’s advice, Gandhiji spent a year travelling around British India, getting to know the land and
its people. His first major public appearance was at the opening of the Banaras Hindu University
(BHU) in February 1916. Further, he was deeply convinced that none of these methods of political
struggle were really viable; the only answer lay in Satyagraha. His reasons for not joining the
existing political organizations are best explained by his belief that he could only join an organization or
a movement that adopted non-violent Satyagraha as its method of struggle. Hence option (b) is the
correct answer.
• When his turn came to speak, Gandhiji charged the Indian elite with a lack of concern for the labouring
poor. Gandhiji’s speech at Banaras in February 1916 was, at one level, merely a statement of fact –
namely, that Indian nationalism was an elite phenomenon, a creation of lawyers and doctors and
landlords. But, at another level, it was also a statement of intent – the first public announcement of
Gandhiji’s own desire to make Indian nationalism more properly representative of the Indian people as a
whole.
• Among the invitees to this event were the princes and philanthropists whose donations had contributed to
the founding of the BHU. Also present were important leaders of the Congress, such as Annie
Besant. Compared to these dignitaries, Gandhiji was relatively unknown. He had been invited on account
of his work in South Africa, rather than his status within India.

24 www.visionias.in ©Vision IAS


Q 69.A
• Among the numerous tribal revolts, the Santhal hool or uprising was the most massive. The Santhals, who
live in the area between Bhagalpur and Rajmahal, known as Daman-i-koh, rose in revolt; made a
determined attempt to expel the outsiders — the dikus — and proclaimed the complete ‘annihilation’ of
the alien regime.
• The Santhals considered the dikus and government servants morally corrupt being given to beggary,
stealing, lying and drunkenness. By 1854, the tribal heads, the majhis and parganites, had begun to meet
and discuss the possibility of revolting. Stray cases of the robbing of zamindars and moneylenders began
to occur.
• The Santhals believed that their actions had the blessings of God. Sido and Kanhu, the principal
rebel leaders, claimed that Thakur (God) had communicated with them and told them to take up
arms and fight for independence. Hence, statement 1 is correct.
• The leaders mobilized the Santhal men and women by organizing huge processions through the villages
accompanied by drummers and other musicians. The leaders rode on horses and elephants. Soon nearly
60,000 Santhals had been mobilized. Forming bands of 1,500 to 2,000, but rallying in many thousands at
the call of drums on particular occasions, they attacked the mahajans and zamindars and their houses,
police stations, railway construction sites, the dak (post) carriers — in fact, all the symbols of diku
exploitation and colonial power. Hence, statement 2 is not correct.
• The Santhal insurrection was helped by a large number of non-tribal and poor dikus. Gwalas
(milkmen) and others helped the rebels with provisions and services; lohars (blacksmiths) accompanied
the rebel bands, keeping their weapons in good shape. Once the Government realized the scale of the
rebellion, it organized a major military campaign against the rebels. It mobilized tens of regiments under
the command of a major general, declared Martial Law in the affected areas and offered rewards of up to
Rs. 10,000 for the capture of various leaders. Hence, statement 3 is correct.

Q 70.B
• SPARSH- System for Pension Administration (Raksha) is an initiative of the Ministry of Defence, to
meet the pension sanctions and disbursement requirement for armed forces vis. Army, Navy,
Airforce and Defence civilians. Hence option (b) is the correct answer
• This web-based system processes pension claims and credits, and pensions directly into bank accounts of
defence pensioners without relying on any external intermediary.
• SPARSH has been designed to keep defence pensioners at the centre, who will be given a completely
transparent view of their pension account, through an online portal.
• SPARSH has fundamentally re-engineered the process of pension disbursement From the generation of
the Pension Payment Orders (PPOs) to the Direct Benefit Transfer of pensions, with the motto of the right
pension delivered at the right time.

Q 71.C
• The Brahmo Samaj had the issue of widow remarriage high on its agenda and did much to popularise it.
• But it was mainly due to the efforts of Pandit Ishwar Chandra Vidyasagar (1820-91), the principal of
Sanskrit College, Calcutta, that the Hindu Widows’ Remarriage Act, 1856, was passed; it legalized the
marriage of widows and declared issues from such marriages as legitimate.
o Vidyasagar cited Vedic texts to prove that the Hindu religion sanctioned widow remarriage.
• Jagannath Shankar Seth and Bhau Daji were among the active promoters of girls’ schools in
Maharashtra.
• Vishnu Shastri Pandit founded the Widow Remarriage Association in the 1850s.
• Karsandas Mulji, a contemporary of Dadabhai Naoroji, was one of the pioneer Indian social reformers
working for the cause of women's emancipation.
o In 1850s-60s, Mulji was a prominent member of the “Bombay intelligentsia” in conflict with the
“merchant aristocracy” over social issues.
o Mulji’s place in Indian history as a reformer is due to the Maharaj libel case in 1862 which earned
him the title of “a Reformer, a Martin Luther of the Banian Cast”.
o He started the Satya Prakash in Gujarati in 1852 to advocate widow remarriage. Hence option
(c) is the correct answer.

25 www.visionias.in ©Vision IAS


Q 72.D
• Recent context: Prime Minister of India Narendra Modi, the Prime Minister of Norway Jonas Gahr
Støre, the Prime Minister of Sweden Magdalena Andersson, the Prime Minister of Iceland Katrín
Jakobsdóttir, the Prime Minister of Finland Sanna Marin, and the Prime Minister of Denmark Mette
Frederiksen held a Summit hosted by the Danish Prime Minister, which is known as the 2nd India-Nordic
Summit.
• The first India-Nordic Summit was held in Stockholm in April 2018. The second summit was
scheduled to take place in June 2021 but was postponed amid Covid-19 fears. The significance of
this event can be assessed from the fact that the United States is the only other country with which
the Nordic countries have summit-level engagement. Hence statement 1 is not correct.
• During the Summit, the Prime Ministers pledged to continue to deepen cooperation between the Nordic
countries and India and focused their discussions on key issues related to international peace and security,
including the conflict in Ukraine, multilateral cooperation, green transition and climate change, the blue
economy, innovation, and digitalization. The Prime Ministers reaffirmed the importance of free trade as a
driver for achieving inclusive growth and realizing the Sustainable Development Goals.
• India and Nordic countries are committed to adopting the ambitious Post 2020 Global Biodiversity
Framework in the upcoming 2nd Part of COP15 of the Convention on Biological Diversity to be held at
Kunming, China, and work together in its implementation.
• India and the Nordic countries emphasized the importance of green technologies and industry transition
with the Leadership Group for Industry Transition (LeadIT) as a notable example. Transport systems,
including aviation knowledge exchange, maritime solutions, and port-modernization were important for
enhancing trade between the Nordic countries and India.
• A geographical and cultural region in Northern Europe and the North Atlantic are Nordic countries
including Finland, Iceland, Norway, Denmark, Sweden, and the Faroe Islands (an archipelago of
islands as an autonomous country within the kingdom of Denmark). Hence statement 3 is not
correct.
• On the sidelines of the summit, Prime Minister Narendra Modi met his counterparts from Denmark,
Norway, Sweden, Iceland, and Finland in a series of bilateral meetings. The highlights of the meetings are
as follows:
o India-Denmark: Discussed a wide range of issues of mutual interest to boost bilateral relations,
including the war in Ukraine, the India-European Union (EU) free trade deal, and the situation in the
Indo-Pacific. Agreed to further strengthen the Green Strategic Partnership with a focus on
green hydrogen, renewable energy, and wastewater management.
 The Green Strategic partnership that builds on and consolidates the existing agreement
establishing a Joint Commission for Cooperation between India and Denmark was signed
and agreed upon by both the countries on 28th September 2020. The partnership is a mutually
beneficial arrangement to advance political cooperation, expand economic relations and green
growth, create jobs, and strengthen cooperation on addressing global challenges and
opportunities; with a focus on an ambitious implementation of the Paris Agreement and the UN
Sustainable Development Goals. Hence statement 2 is not correct.

26 www.visionias.in ©Vision IAS


Q 73.B
• Both the Zamindari and Ryotwari systems departed fundamentally from the traditional land systems of the
country. The British created a new form of private property in land in such a way that the benefit of the
innovation did not go to the cultivators.
• All over the country land was now made salable, mortgageable, and alienable. This was done
primarily to protect the Government's revenue.
• If the land had not been made transferable or salable, the Government would find it very difficult
to realize revenue from a cultivator who had no savings or possessions out of which to pay it. Now
he could borrow money on the security of his land or even all part of it and pay his land revenue. If
he refused to do so, the Government could and often did auction his land and realize the amount. Hence
option (b) is the correct answer.
• The British by making land a commodity that could be freely bought and sold introduced a fundamental
change in the existing land systems of the country. Due to this stability and the continuity of the Indian
villages were shaken.

Q 74.B
• The revolutionaries in northern India were the first to emerge out of the mood of frustration and
reorganize under the leadership of the old veterans, Ramprasad Bismil, Jogesh Chatterjea and
Sachindranath Sanyal whose Bandi Jiwan served as a textbook for the revolutionary movement. Hence
pair 1 is not correctly matched.
• The most spectacular revolutionary terrorist actions of the period were the unsuccessful attempt under the
leadership of Rash Behari Bose and Sachindranath Sanyal to kill the Viceroy, Lord Hardinge who was
wounded by the bomb thrown at him while he was riding an elephant in a state procession.
• The revolutionaries Sukhdev, Bhagwati Charan Vohra, Shiv Varma, Bejoy Sinha, Yashpal, all were
intellectuals of a high order. The draft of the famous statement of revolutionary position, The Philosophy
of the Bomb, was written by Bhagwati Charan Vohra at the instance of Azad and after a full
discussion with him. Hence pair 2 is not correctly matched.
• Bhagat Singh turned to Marxism and had come to believe that popular broad-based mass movements
alone could lead to a successful revolution; in other words revolution could only be achieved ‘by the
masses for the masses.’ That is why Bhagat Singh helped establish the Punjab Naujawan Bharat Sabha in
1926 (becoming its founding Secretary), as the open wing of the revolutionaries. Why I Am an Atheist is
an essay written by Bhagat Singh in 1930 in Lahore Central Jail. Hence pair 3 is correctly matched.

Q 75.A
• There was a gradual erosion in the power of the Mughals, it caused an eclipse of various cities associated
with their rule. Delhi and Agra which were the capitals during the Mughal rule lost their political authority
and grandeur.
• The emergence of urban centres brought many changes in the network of trade. For example, the
Portuguese settled in Panaji in 1510 and the Dutch in Masulipatnam in 1605. The British came to Madras
in 1639.
• During the 18th century, many new regional capitals emerged. They soon gained importance. Such
powers were Lucknow, Hyderabad, Seringapatam, Poona (Pune), Nagpur, Baroda (Vadodara) and
Thanjavur (Tanjore).
• In the mid-eighteenth century, commercial centres like Surat, Dhaka and Masulipatnam lost their
importance. With the British becoming more powerful after the Battle of Plassey in 1757, the trade
shifted to cities like Madras, Calcutta and Bombay which had emerged as new economic capitals
due to the trade activities of the East India Company. So these cities also became centres of colonial,
political and administrative power. By the end of 1800, Madras, Calcutta and Bombay had become the
biggest cities in India. Hence option (a) is the correct answer.
• By the end of the eighteenth century, the land-based empires in Asia were replaced by the powerful
sea-based European empires. It ushered in International trade, mercantilism and capitalism in the
society.

Q 76.B
• Recently Kerala health department sought to contain the Shigella spread as it is believed to be the reason
behind the incident of suspected food poisoning in Kasaragod district.
• Shigella is a genus of bacteria that causes an infection called shigellosis.

27 www.visionias.in ©Vision IAS


o Shigellosis is a gastrointestinal infection caused by one of four species of Shigella bacteria, including
S. sonnei.
• It is the second leading cause of bacterial diarrhea worldwide and the third leading cause of death in
children less than 5 years old.
• Spread: Transmitted through contaminated food or water, or through person-to-person contact. It
is endemic in most low- or middle-income countries.
• No vaccines are available for shigellosis.
• Hence option (b) is the correct answer.

Q 77.C
• In Tripuri Session of 1939, Subhas Chandra Bose defeated Pattabhi Sitaramayya, who was
nominated by Gandhiji. After which the INC faced an internal crisis where Subhas faced ideological
difference between many other leaders and also confronted with Gandhiji regarding to the initiation of a
mass struggle against the British. Subhas Bose believed that the Congress was strong enough to start
an immediate struggle, whereas Gandhiji said that the time was not right for a mass struggle.
• Govind Ballabh Pant moved a resolution at Tripuri expressing full confidence in the old Working
committee, reiterating full faith in Gandhiji’s leadership of the movement and the Congress policies and
asking Subhas Bose to nominate his Working Committee ‘in accordance with the wishes of Gandhiji. The
resolution was passed by a big majority, but Gandhiji did not approve of the resolution and refused to
impose a Working Committee on Subhas. He asked him to nominate a Committee of his own
choice. Hence statement 1 is correct.
• Bose continued his effort to win Gandhi’s confidence but did not succeed. Bose refused to nominate a
new working committee. In the circumstances, Bose saw no option but to resign. He resigned from the
president’s post in April 1939 after which he started his own party ‘Forward block’ within the
Congress. Rajendra Prasad was elected as the President of INC after it. Hence statement 2 is
correct.

Q 78.D
• The struggle against untouchability and for the social and economic uplift of the depressed classes
continued all over India after 1924 as a part of the Gandhian constructive programme. Struggle was most
Intense m Kerala. Prodded by K Kelappan, the Kerala Pradesh Congress Committee (KPCC) took up
the question of temple entry in 1931 during the period when the Civil Disobedience Movement was
suspended. A vast campaign of public meetings was organized throughout Malabar. The KPCC decided to
make a beginning by organizing a temple entry Satyagraha at Guruvayur on 1st November 1931.
• A jatha of sixteen volunteers, led by the poet Subramanian Tirumambu, who became famous as the
‘Singing Sword of Kerala,’ began a march from Cannanore in the north to Guruvayur on 21
October.
• The volunteers ranged from the lowliest of Harijans to the highest caste Namboodiris. The march stirred
the entire country and aroused anti-caste sentiments. The 1st of November was enthusiastically observed
as All-Kerala Temple Entry Day with a programme of prayers, processions, meetings, receptions and fund
collections.
• The anti-untouchability movement gained great popularity. Many religious devotees transferred the
offerings they would have made to the temple to the Satyagraha camp, feeling that the camp was even
more sacred than the temple.
• The temple authorities also made arrangements. They put up barbed wire all around the temple and
organized gangs of watchmen to keep the Satyagrahis out and to threaten them with beating.
• On 1 November, sixteen white khadi-clad volunteers marched to the eastern gate of the temple where their
way was barred by a posse of policemen headed by the Superintendent of Police. Very soon, the temple
servants and local reactionaries began to use physical force against the peaceful and non-violent
Satyagrahis while the police stood by.
• For example, P Krishna Pillai and A.K. Gopalan, who were to emerge later as major leaders of the
Communist movement in Kerala, were mercilessly beaten. The Satyagraha continued even after the
Civil Disobedience Movement was resumed in January 1932 and all Congress Committees were
declared unlawful and most of the Congressmen leading the Satyagraha were imprisoned.
• The Satyagraha entered a new phase on 21 September 1932 when K. Kelappan went on a fast unto
death before the temple until it was opened to the depressed classes. The entire country was again
stirred to its depths. Once again meetings and processions engulfed Kerala and many other parts of the

28 www.visionias.in ©Vision IAS


country. Caste Hindus from Kerala as well as the rest of India made appeals to the Zamorin of Calicut,
custodian of the temple, to throw open the temples to all Hindus; but without any success.
• Gandhiji made repeated appeals to Kelappan to break his fast, at least temporarily, with an assurance that
he would himself, if necessary, undertake the task of getting the temple opened. Finally, Kelappan broke
his fast on October 2, 1932. The Satyagraha was also suspended. But the temple entry campaign was
carried on ever more vigorously.
• Even though the Guruvayur temple was not opened immediately, the Satyagraha was a great success in
broader terms.
• Hence, option (d) is the correct answer.

Q 79.A
• In 1878, Keshab’s inexplicable act of getting his thirteen-year-old daughter married to the minor
Hindu Maharaja of Cooch-Behar with all the orthodox Hindu rituals caused another split in
Keshab’s Brahmo Samaj of India.
• Earlier, Keshab had begun to be considered an incarnation by some of his followers, much to the dislike
of his progressive followers. Further, Keshab had begun to be accused of authoritarianism.
• After 1878, the disgusted followers of Keshab set up a new organization, the Sadharan Brahmo
Samaj.
• The Sadharan Brahmo Samaj was started by Ananda Mohan Bose, Shibchandra Deb, and Umesh
Chandra Datta. Hence statement 1 is correct.
• It reiterated the Brahmo doctrines of faith in a Supreme being, one God, the belief that no scripture
or man is infallible, and belief in the dictates of reason, truth, and morality. Hence statement 2 is not
correct.

Q 80.B
• Alfonso de Albuquerque served as viceroy of Portuguese India from 1509 to 1515. He was the real
founder of the Portuguese power in the East, a task he completed before his death. He secured for
Portugal the strategic control of the Indian Ocean by establishing bases overlooking all the entrances to
the sea.
• There were Portuguese strongholds in East Africa, off the Red Sea, at Ormuz; in Malabar; and at
Malacca. The Portuguese, under Albuquerque, bolstered their stranglehold by introducing a permit
system for other ships and exercising control over the major ship-building centers in the region. The
non-availability of timber in the Gulf and Red Sea regions for ship-building also helped the Portuguese in
their objectives.
• Albuquerque acquired Goa from the Sultan of Bijapur in 1510 with ease; the principal port of the
Sultan of Bijapur became “the first bit of Indian territory to be under the Europeans since the time
of Alexander the Great”. An interesting feature of his rule was the abolition of sati. Hence option (b) is
the correct answer.

Q 81.D
• The East India Company in order to promote British manufacturers imposed a deliberate policy of free
trade which was completely one sided. While the doors of India were open to foreign goods, Indian goods
were subjected to high import duties in Britain. Also Indian hand made goods were unable to compete
against the much cheaper prodcuts of British mills.
• These policies forced India to export raw materials like cotton and silk instead of manufactured products.
• Thus by the end of 19th century, Indian exports consisted primarily of raw cotton, jute, silk,
oilseeds, wheat, hides and skins, indigo and tea
• Hence, option (d) is the correct answer.

Q 82.B
• Recent context: The 12th meeting of Interpol’s Match-Fixing Task Force (IMFTF) called for
harmonized global efforts to curb competition manipulation and the establishment of national
platforms, as outlined by Macolin Convention.
• It was the first major event under the newly created Financial Crime and Anti-corruption Centre
(IFCACC) from Interpol. CBI from India participated in the event.
• The Council of Europe Convention on the Manipulation of Sports Competitions, known as the
Macolin Convention, is a multilateral treaty entered into force on 1 September 2019 and aimed at
checking match-fixing. Hence option (b) is the correct answer.
29 www.visionias.in ©Vision IAS
Q 83.B
• Recent context: President conferred 13 Shaurya Chakras, including six posthumous, to the personnel of
the Armed Forces during Defence Investiture Ceremony.
• The award was Instituted on 4 Jan 1952 as Ashoka Chakra Class-III and renamed on 27 Jan 1967 as
Shaurya Chakra and awarded for gallantry, otherwise than in the face of the enemy. The decoration
may be awarded posthumously.
• The Shaurya Chakra is India's third-highest peacetime gallantry award after the Ashok Chakra
and Kirti Chakra. Hence statement 1 is not correct.
• The following categories of personnel shall be eligible for the award of Shaurya Chakra:
 Officers, men and women of all ranks of the Army, the Navy and the Air Force, of any of the Reserve
Forces, of the Territorial Army, Militia, and of any other lawfully constituted forces.
 Members of the Nursing Services of the Armed Forces.
 Civilian citizens of either sex in all walks of life and members of Police forces including Central
Para-Military Forces and Railway Protection Force. Hence statement 2 is correct.

Q 84.A
• During the 1920s in South India, the non-brahmins organized the Self-Respect Movement led by E.V.
Ramaswamy Naicker.
• There were numerous other movements demanding that the ban on the entry of lower castes into temples
be lifted. Sri Narayana Guru in Kerala led a lifelong struggle against upper caste domination.
• Sri Narayana Guru coined the slogan “one religion, one caste, one God for mankind”, which his
disciple Sahadaran Ayyapan changed into “no religion, no caste, no God for mankind”.
• Hence option (a) is the correct answer.

Q 85.B
• The growing nationalist feeling in the country and the urge for national unity produced two historic
developments at the Lucknow session of the India National Congress in 1916.
o Firstly, the two wings of the Congress were reunited. The old controversies had lost their meaning and
the split in the Congress had led to political inactivity, The Lucknow Congress was the first united
Congress since 1907. Tilak and his men were welcomed back into the Congress by the Moderate
president, Ambika Charan Mazumdar of Lucknow session (1916). Hence statement 3 is correct.
o Efforts of Annie Besant and Tilak meet with success at the annual session of the Congress in
December 1915 it was decided that the Extremists be allowed to rejoin the Congress. The
opposition from the Bombay group had been greatly weakened by the death of Pherozeshah
Mehta. Hence statement 2 is correct.
o Secondly, at Lucknow, the Congress and the All India Muslim League sank their old differences and
put up common political demands before the Government. The unity between the Congress and the
League was brought about by the signing of the Congress League Pact, known popularly as the
Lucknow Pact. The Lucknow Pact marked an important step forward in Hindu-Muslim unity.
o Both Tilak and Annie Besant had played a leading role in bringing about this agreement
between the Congress and the League, much against the wishes of many important leaders,
including Madan Mohan Malaviya. Hence statement 1 is not correct.
Q 86.B
• Events during Lytton's Viceroyalty from 1876-80:
o Most of the import duties on British textile imports were removed to please the textile
manufacturers of Britain.
o Famine of 1876-78 affected Madras, Bombay, Mysore, Hyderabad, parts of central India, and Punjab;
appointment of the Famine Commission under the presidency of Richard Strachey (1878).
o Royal Titles Act (1876), Queen Victoria assumed the title of ‘Kaiser-i-Hind’ or Queen Empress of
India.
o The Second War against Afghanistan aroused vehement agitation against the heavy cost of this
imperialist war, which the Indian Treasury was made to bear.
o The Arms Act of 1878, which disarmed the people, appeared to them as an effort to emasculate
the entire nation.
o The Vernacular Press Act of 1878 was condemned by the politically conscious Indians as an
attempt to suppress the growing nationalist criticism of the alien government.
o The holding of the imperial Durbar at Delhi for the first time in 1877 at a time when the country
was suffering from a terrible famine led people to believe that their rulers cared very little even

30 www.visionias.in ©Vision IAS


for their lives. The Delhi Durbar was an Indian imperial style mass assembly organized by the British
in Delhi, to mark the succession of an Emperor or Empress of India. Also known as the Imperial
Durbar, it was held three times, in 1877, 1903, and 1911, at the height of the British Empire.
o In 1878, the government announced new regulations reducing the maximum age limit for sitting in the
Indian Civil Service Examination from 21 years to 19.
o Lytton’s viceroyalty helped to intensify discontent against foreign rule.
o In 1883, Ripon who succeeded Lytton as the Viceroy tried to pass a law to enable Indian district
magistrates and session judges to trial Europeans in criminal cases. Under the existing law, even
Indian members of the Indian Civil Service were not authorized to try Europeans in their courts.
o The Europeans in India organized a vehement agitation against this Bill, which came to be known as
Ibert Bill (after Ilbert, the Law Member). The Ilbert Bill poured abuse on Indians and their culture and
character. They declared that even the most highly educated among the Indians were unfit to trial a
European.
• Hence option (b) is the correct answer.

Q 87.C
• On 1 November 1913, the first issue of Ghadar, in Urdu was published, and on 9 December, the
Gurmukhi edition. The name of the paper left no doubts as to its aim. Ghadar means Revolt. And if any
doubts remained, they were to be dispelled by the captions on the masthead: ‘Angrezi Raj ka Dushman’ or
‘An Enemy of British Rule.’ On the front page of each issue was a feature titled Angrezi Raj Ka Kacha
Chittha or ‘An Expose of British Rule.’ Hence statement 1 is correct.
• Finally, in 1914, three events influenced the course of the Ghadar movement: the arrest and escape
of Har Dayal, the Komagata Maru incident, and the outbreak of the First World War. Hence
statement 2 is not correct.
• March 1914, the ship, Komagata Maru had begun its fateful voyage to Canada. Canada had for some
rears imposed very strict restrictions on Indian immigration by means of a law that forbade entry to all,
except those who made a continuous journey from India.
• When the ship arrived in Vancouver, it was not allowed into the port and was cordoned off by the
police. To fight for the rights of the passengers, ‘Shore Committee was set up under the leadership
of Husain Rahim, Sohan Lal Pathak, and Balwant Singh, funds were raised, and protest meetings
were organized. Hence statement 3 is correct.
• Soon the Komagata Maru was forced out of Canadian waters. On landing at Budge Budge near Calcutta,
the harassed and irate passengers, provoked by the hostile attitude of the authorities, resisted the police
and this led to a clash in which eighteen passengers were killed, and 202 arrested. A few of them
succeeded in escaping

Q 88.D
• Pherozeshah Mehta retired from the Imperial Legislative Council in 1901 due to bad health. Gopal
Krishna Gokhale got elected in his place, a thirty-five-year-old who had already made his mark as
the Secretary of the Poona Sarvajanik Sabha and the editor of the Sudharak. In 1897, as a witness in
London before the Royal Commission on Expenditure in India, Gokhale had outshone veterans like
Surendranath Banerjea, D.E. Wacha, G. Subramaniya Iyer, and Dadabhai Naoroji. Gokhale was to prove a
more than worthy successor to Mehta.
• Gopal Krishna Gokhale was an outstanding intellectual who had been carefully trained in Indian
economics by Justice Ranade and G.V. Joshi.
• Gokhale gained great fame for his budget speeches which used to be reported extensively by the
newspapers and whose readers would wait eagerly for their morning copy. He was to transform the
Legislative Council into an open university for imparting political education to the people.
• His very first budget speech on 26 March 1902 established him as the greatest parliamentarian that
India could produce.
• Gandhiji declared him his political guru. And Balgangadhar Tilak, his lifelong political opponent, said at
his funeral: ‘This diamond of India, this jewel of Maharashtra, this prince of workers, is taking eternal rest
on the funeral ground. Look at him and try to emulate him.”
• Hence option (d) is the correct answer.

31 www.visionias.in ©Vision IAS


Q 89.D
• The Indian Association of Calcutta (also known as the Indian National Association) superseded the
Indian League and was founded in 1876 by younger nationalists of Bengal led by Surendranath
Banerjea and Ananda Mohan Bose, who were getting discontented with the conservative and pro-
landlord policies of the British Indian Association.
• The Indian Association was the most important of pre-congress associations and aimed to “promote by
every legitimate means the political, intellectual and material advancement of the people.”
• It set out to—
o create a strong public opinion on political questions, and
o unify Indian people in a common political program.
• It protested against the reduction of the age limit in 1877 for candidates of the Indian Civil Service
examination. Hence statement 1 is correct.
• The association demanded simultaneous holding of civil service examinations in England and India
and Indianisation of higher administrative posts. Hence statement 2 is correct.
• It led a campaign against the repressive arms act (1878) and the vernacular press act. Hence
statement 3 is not correct.

Q 90.D
• Recent Context: With a view to conserving water for the future, the Prime Minister launched a new
Mission on Amrit Sarovar.
• The Mission is aimed at developing and rejuvenating 75 water bodies in each district of the country
as a part of the celebration of Azadi ka Amrit Mahotsav. In total, it would lead to the creation of
50,000 water bodies of a size of about an Acre or more. Hence statement 1 is not correct
• This Mission has been launched with a whole Government Approach in which 6
Ministries/Department namely Dept of Rural Development, Department of land resources,
Department of Drinking Water and Sanitation, Department of Water Resources, Ministry of
Panchayati Raj, Ministry of Forest, Environment and Climate changes. Also, Bhaskaracharya
National Institute for Space Application and Geo-informatics(BISAG-N) has been engaged as a
Technical Partner for the Mission. Hence statement 2 is not correct
• The Mission Amrit Sarovar is to be completed by 15th August 2023. People’s participation in the Mission
is the focal point. Local freedom fighters, their family members, Martyr’s family members, Padma
Awardee, and citizens of the local area wherein an Amrit Sarovar is to be constructed, will be engaged at
all stages. On the 15th of August, National Flag hoisting will be organized on every Amrit Sarovar site.

Q 91.C
• An alternative policy of weakening the nationalist movement was started by the British. Instead of
sneering at the Moderates, the policy was to be that of ‘rallying’ them as John Morley, the new Secretary
of State for India, put it in 1907. The new policy is known as the policy of the carrot and the stick. It may
be described as a policy of repression-conciliation-suppression. The entire objective of the new policy was
to isolate the Extremists. Hence option (a) is correct.
• The Government of India, headed by Lord Minto as Viceroy and John Morley as the Secretary of State,
offered a bait of fresh reforms in the Legislative Councils and in the beginning of 1906 began discussing
them with the Moderate leadership of the Congress. The Moderates agreed to cooperate with the
Government and discuss reforms even while a vigorous popular movement, which the Government was
trying to suppress, was going on in the country. The result was a total split in the nationalist ranks. Hence
option (b) is correct.
• The main public leaders of the two wings, Bal Gangadhar Tilak (of the Extremists) and Gokhale (of the
Moderates) were mature politicians who had a clear grasp of the dangers of disunity in the nationalist
ranks. When it came to the crunch, Tilak had to go with the more extreme leaders like Aurobindo
Ghosh. Hence option (c) is not correct.
• Matters nearly came to a head at the Calcutta Congress in 1906 over the question of its Presidentship. A
split was avoided by choosing Dadabhai Naoroji, who was respected by all the nationalists as a great
patriot. Four compromise resolutions on the Swadeshi, Boycott, National Education, and Self-
Government demands were passed. Hence option (d) is correct.

32 www.visionias.in ©Vision IAS


Q 92.A
• The Battle of Miani between a British force of about 2,800 troops under Sir Charles Napier and a
host of more than 20,000 followers of the amirs (chiefs) of Sindh ended in a British victory and the
annexation of most of Sindh.
o Complaints had been made against the amirs’ attitude toward the British during the First Anglo-
Afghan War (1839–42). Instead of leaving the settlement to the British resident, the British gave full
civil and military powers to Napier in September 1842.
o Napier forced on the amirs an onerous new treaty and provocatively seized and razed the desert
fortress of Imamgarh. A popular upsurge then led to open war. At Miani the British prevailed.
The army of the amirs was scattered, and Sindh was annexed in February 17, 1843.
• Sikh Wars, (1845–46; 1848–49), two campaigns fought between the Sikhs and the British. They resulted
in the conquest and annexation by the British of Punjab in northwestern India.
o The first war was precipitated by mutual suspicions and the turbulence of the Sikh army. The Sikh
state in the Punjab had been built into a formidable power by the maharaja Ranjit Singh, who ruled
from 1801 to 1839. Within six years of his death, however, the government had broken down in a
series of palace revolutions and assassinations.
o By 1843 the ruler was a boy—the youngest son of Ranjit Singh—whose mother was proclaimed
queen regent. Actual power, however, resided with the army, which was itself in the hands of panchs,
or military committees. Relations with the British had already been strained by the refusal of the Sikhs
to allow the passage of British troops through their territory during the First Anglo-Afghan War
(1838–42).
o Having determined to invade British India under the pretext of forestalling a British attack, the Sikhs
crossed the Sutlej River in December 1845. They were defeated in the four bloody and hard-fought
battles of Mudki, Firozpur, Aliwal, and Sobraon. The British annexed Sikh lands east of the Sutlej and
between it and the Beas River; Kashmir and Jammu were detached, and the Sikh army was limited to
20,000 infantry and 12,000 cavalry. A British resident was stationed in Lahore with British troops.
o The Second Sikh War began with the revolt of Mulraj, governor of Multan, in April 1848 and
became a national revolt when the Sikh army joined the rebels on September 14. Indecisive
battles characterized by great ferocity and bad generalship were fought at Ramnagar
(November 22) and at Chilianwala (Jan. 13, 1849) before the final British victory at Gujrat
(February 21). The Sikh army surrendered on March 12, 1848, and Punjab was then annexed.
• To bring all the native states under the direct British rule, the then Governor-General of India, Lord
Dalhousie (1848-1856) enunciated a policy known as the 'Doctrine of Lapse'. Under this policy, if the
ruler of any princely state died without a natural heir, his adopted son would not be recognized as his
successor by the British and that native state lapsed to the British dominion.
o By the application of this policy, a number of Indian states were brought under the British sway. It
was also applied to Jhansi when its ruler Raja Gangadhar Rao died without a natural heir.
o On the death of her husband, Rani Lakshmi Bai declared their adopted son Anand Rao as the
successor of the late Raja. However the then Governor-General of India had already enunciated
'Doctrine of Lapse'. By the application of this policy, Dalhousie annexed Satara, Jaipur,
Tanjore, Sambalpur, etc. to the British dominion. So Dalhousie refused to accept Anand Rao as
the successor of the late Gangadhar Rao and annexed Jhansi in 1853.
• Hence, option (a) is the correct answer.

Q 93.D
• Recently, recoveries by financial creditors under IBC shrink to 33%. It was 39.3 percent as of March
2021, and as high as 46 percent till March 2020, according to the Insolvency and Bankruptcy Board of
India (IBBI) data.
• About financial creditors (FCs)
o It is defined under IBC 2016 as a creditor to whom a financial debt such as a loan/debt security is
owed by the corporate debtor. Hence statement 1 is not correct.
 Whereas an operational creditor (OCs) is a creditor who has provided goods or services to the
corporate debtor, including employees, and payable to central or state governments.
o Any person whether he/she is a financial creditor, operational creditor or the corporate debtor
itself may initiate corporate insolvency resolution. Hence statement 2 is not correct.
o FC(s) is/are part of the Committees of Creditors (CoC).
 CoC is mandated under IBC and empowered to take key decisions, including decisions on
haircuts (the total claims minus the amount of realization/amount of the claims) for creditors,
that are binding on all stakeholders, including those dissenting.
33 www.visionias.in ©Vision IAS
▪ Key decisions by CoC is taken through the voting process.
• In the event of a default, a financial creditor may collectively or separately with other lenders file an
application for the onset of arbitration proceedings before an adjudicating officer.

Q 94.A
• Gopal Krishna Gokhale (1866-1915), a liberal leader of the Indian National Congress, founded the
Servants of India Society in 1905 with the help of M.G. Ranade. In 1911, the Hitavada began to be
published to project the views of society. Society chose to remain aloof from political activities and
organizations like the Indian National Congress. Hence, pair 1 is correctly matched.
• Gopal Hari Deshmukh (1823-1892) was a social reformer and rationalist from Maharashtra. He held the
post of a judge under the British raj but wrote for a weekly Prabhakar under the pen name of
Lokahitawadi on social reform issues. He attacked Hindu orthodoxy and supported social and religious
equality. He wrote against the evils of the caste system. He said, “If religion does not sanction social
reform, then change religion.” He started a weekly, Hitechhu, and also played a leading role in
founding the periodicals, Gyan Prakash, Indu Prakash, and Lokahitawadi. Hence, pair 2 is not
correctly matched.
• Gopal Ganesh Agarkar (1856-1895) was an educationist and social reformer from Maharashtra. He was
a co-founder of the New English School, the Deccan Education Society, and Fergusson College. He was a
principal of Fergusson College. He was also the first editor of Kesari, the journal started by Lokmanya
Tilak. Later, he started his own periodical, Sudharak, which spoke against untouchability and the
caste system. Hence, pair 3 is not correctly matched.

Q 95.C
• Surendranath Banerjea and many other leaders of Bengal had not attended the first session of the
National Congress as they were busy with the Second National Conference at Calcutta.
o In 1886, Surendranath Banerjea and other leaders of Bengal merged their forces with those of the
National Congress whose second session met in Calcutta in December 1886 under the president-ship
of Dadabhai Naoroji. Hence statement 1 is not correct and 2 is correct.
o From the Calcutta session, the National Congress became ‘the whole country's Congress’. Its
delegates, numbering 436, were elected by different local organizations and groups.
• In 1890, Kadambini Ganguli, the first woman graduate of Calcutta University addressed the 6th Congress
session. Hence statement 3 is not correct.

Q 96.D
• The major changes introduced after 1857 were:
o The Army Amalgamation Scheme, 1861 moved the Company’s European troops to the services
of the Crown. Hence statement 1 is correct.
o Further, the European troops in India were constantly revamped by periodical visits to England,
sometimes termed the ‘linked-battalion’ scheme. The proportion of Europeans to Indians in the
army was raised and fixed at one to two in the Bengal army and two to five in the Madras and
Bombay armies. Hence statement 2 is correct.
o All Indian artillery units, except a few mountain batteries, were made defunct.
o All higher posts in the army and the artillery departments were reserved for the Europeans.
Hence statement 3 is correct.
o Until the first decade of the twentieth century, no Indian was considered fit to deserve the king’s
commission. A new English recruit was considered superior to an Indian officer holding the viceroy’s
commission.
o In the wake of the transfer of power from the British East India Company to the British Crown,
a section of European forces employed under the Company resented the move that required the
three Presidency Armies to transfer their allegiance from the defunct Company to the Queen, as
in the British Army. This resentment resulted in some unrest termed White Mutiny.

Q 97.D
• The Quit India Movement marked a new high in terms of popular participation in the national movement
and sympathy with the national cause in earlier mass struggles, the youth were in the forefront of the
struggle.
• Students from colleges and even schools were the most visible element, especially in the early days of
August. Women especially, college and school girls, played a very important role.

34 www.visionias.in ©Vision IAS


• Aruna Asaf Ali and Sucheta Kripalani were two major women organizers of the underground, and
Usha Mehta an important member of the small group that ran the Congress Radio.
• Workers were prominent as well, and made considerable sacrifice by enduring long strikes and braving
police repression in the streets.
• Sucheta Kripalani was born in Ambala (Punjab) in June 1908. Kripalani began her professional career as
a teacher of Constitutional History at Banaras Hindu University till 1939.
o In 1936, she married Acharya Kripalani, a Congress party member and a prominent freedom fighter.
She became a member of the Congress Party in 1938 and served as the Secretary, for one and half
years, to the Foreign Department and Women's Section.
o Kripalani actively participated in the Indian independence movement in the 1940s. That included her
involvement in the 1942 Quit India movement- she evaded arrest from the Government, although she
was eventually arrested in 1944 and was detained for a year.
• Aruna Asaf Ali was a legendary heroine of India's freedom struggle. Her moment of reckoning came in
1942 during Quit India Movement and she rose to the occasion. She unfurled the national flag at the
Gowalia Tank maidan to signify the commencement of the Quit India Movement and became a legend for
thousands of youth that rose to emulate her.
o Her first major political action was during the Salt Satyagraha in 1930 when she addressed public
meetings and led processions. British Government charged her for being a “vagrant” and sentenced
her to one year’s imprisonment.
• Usha Mehta hailed from the village of Saras, situated near Surat in the state of Gujrat. From a young age,
she found herself to be an active participant in the fight against British rule.
o In 1928, at the age of eight, she took part in her first protest against the Simon Commission.
o In 1942, in an All India Congress Committee (AICC) session, Dr. Usha Mehta was privy to the
powerful speeches given by Mahatma Gandhi, Jawaharlal Nehru, Maulana Azad and Sardar
Vallabhbhai Patel.
o This gave her the impetus to organise a secret radio station and help deliver news to the remotest
corners of the world. She ardently believed that the radio could be used as tool to help deliver facts
and inform the people of the world about the events taking place in her country. This, she believed,
would also give us the opportunity to put forth our side of the story, as against that of the colonizer’s.
• Hence, option (d) is the correct answer.

Q 98.C
• In 1866, Dadabhai Naoroji organized the East India Association in London to discuss the Indian
question and to influence British public men to promote Indian welfare. Later he organized branches of
the Association in prominent Indian cities. Hence statement 1 is correct.
• Born in 1825, Dadabhaj devoted his entire life to the national movement and soon came to be known as
the Grand Old Man of India.
• He was also India’s first economic thinker. In his book Poverty and Un-British Rule on economics, he
showed that the basic cause of India’s poverty lay in the British exploitation of India and the drain of its
wealth.
• Dadabhai was honored by being thrice elected president of the Indian National Congress in 1886,
1893, and 1906. Hence statement 2 is correct.
• While until the end of the 19th century, Indian nationalists confined their political demands to a share in
political power and control over the purse, by 1905 most of the prominent nationalists were putting
forward the demand for some form of self-government. Here again, Dadabhai Naoroji was the most
advanced.
• Speaking on the drain at the International Socialist Congress in 1904, he put forward the demand for ‘self-
government’ and treatment of India ‘like other British Colonies.” Hence statement 3 is not correct.
o A year later in 1905, in a message to the Benares session of the Indian National Congress, Dadabhai
categorically asserted: ‘Self-government is the only remedy for India’s woes and wrongs.’
o And, then, as the President of the 1906 session of the Congress at Calcutta, he laid down the goal of
the national movement as “self-government or Swaraj,” like that of the United Kingdom or the
Colonies.’

Q 99.D
• In Bengal, too, the revolutionary terrorists started reorganizing and developing their underground
activities. At the same time, many of them continued to work in the Congress organization. Among the
new Revolt Groups, the most active and famous was the Chittagong group led by Surya Sen. Surya
35 www.visionias.in ©Vision IAS
Sen gathered around himself a large band of revolutionary youth including Anant Singh, Ganesh Ghosh,
and Lokenath Baul. They decided to organize a rebellion, on however small a scale, to demonstrate that
it was possible to challenge the armed might of the British empire in India. Hence options 1 and 3 are
correct.
• The group had planned to occupy two main armories in Chittagong to seize and supply arms to the
revolutionaries to destroy telephone and telegraph lines and to dislocate the railway link between
Chittagong with the rest of Bengal. The raid was conducted in April 1930 and involved 65 activists under
the banner of the Indian Republican Army - Chittagong Branch. The raid was successful.
• There was large-scale participation of young women especially under Surya Sen. These women provided
shelter, carried messages, and fought with guns in hand. Prominent women revolutionaries in Bengal
during this phase included Pritilata Waddedar, who died conducting a raid; Kalpana Dutt who was
arrested and tried along with Surya Sen and given a life sentence; Santi Ghosh and Suniti
Chandheri, school girls of Comilla, who shot dead the district magistrate. (December 1931); and Bina
Das who fired point-blank at the governor while receiving her degree at the convocation (February
1932). Hence options 2 and 4 are correct.
• Compared to the old revolutionary terrorists, as also Bhagat Singh and his comrades, the Chittagong
rebels made an important advance. Instead of an individual’s act of heroism or the assassination of an
individual, theirs was a group action aimed at the organs of the colonial state. The Chittagong IRA
cadre included many Muslims like Sattar, Mir Ahmad, Fakir Ahmad Mian, and Tunu Mian and got
massive support from Muslim villagers around Chittagong.

Q 100.C
• The nation has witnessed a landmark event in the history of indigenous warship building wherein 2
frontline ships, INS-Surat, a Project 15B Destroyer, and INS-Udaygiri, a Project 17A Frigate of the
Indian Navy were launched at Mazgaon Docks Limited, Mumbai.
• The Project 15B class of ships are the next-generation stealth guided-missile destroyers of the Indian
Navy being built at the Mazgaon Docks Ltd, Mumbai. ‘Surat’ is the fourth ship of Project 15B
Destroyers which heralds a significant makeover of the P15A (Kolkata Class) Destroyers and is
named after the commercial capital of the state of Gujarat and also the second-largest commercial
hub of western India after Mumbai. The ship Surat has been built using the Block construction
methodology which involved hull construction at two different geographical locations and has been joined
together at MDL, Mumbai. The first ship of this class was commissioned in 2021. The second and third
ships have been launched and are at different stages of outfitting/ trials.
• ‘Udaygiri’, named after a mountain range in the state of Andhra Pradesh, is the third ship of
Project 17A Frigates. These follow on of the P17 Frigates (Shivalik Class) with improved stealth
features, advanced weapons and sensors, and platform management systems. ‘Udaygiri’ is the
reincarnation of erstwhile ‘Udaygiri’, the Leander Class ASW Frigate, which saw numerous challenging
operations in its illustrious service to the country spanning over three decades from 18 Feb 1976 to 24
Aug 2007. Hence option (c) is the correct answer.
• Both 15B and P17A ships have been designed in-house by the Directorate of Naval Design (DND), which
has been the fountainhead for all warship design activities of the Nation and during the building phase at
the Shipyard, around 75% of the orders for equipment and systems have been placed on indigenous
firms including MSMEs which is a true testament of ‘Atmanirbharta’ in the country.

36 www.visionias.in ©Vision IAS

You might also like